Вы находитесь на странице: 1из 100

?

Online PG Nepal
Questions Answers Explanations
[QAE file]

CEE MD/MS BASED MODEL TEST Designed to make you more


confident.
Kartik 5, 2077 (Wednesday) Master
Master the high yield and most
frequently tested questions. Simulate
the real test in expected pattern.

Review your Rank among others, and


re-
re-design the extra efforts required
to ace the competitive exams.

Discover your weakness and focus on


it.

Practice and Learn.

Designed by 90+ experts, and near all


toppers and rankers of past 3 years
of competitive exams.

Recommended by all toppers and


rankers of Last year entrance in their
interview.

Contact details:
Email: onlinepgnepal@gmail.com
Facebook: fb.com/onlinepgnepal
This PDF file has been individualized for digital uniqueness based upon your Email address and Mobile Instagram: instagram.com/onlinepgnepal
number. Materials sent by the Online PG Nepal is for educational purpose only. Circulating the contents to anyone
other than the intended by Online PG Nepal in any form (print or digital) would be considered violation of copyright Viber: 9801114177
protection and would attract legal actions. In
n view of the possibility of human error or changes in medical sciences, Mobile: 9801114177
we don't warrant that the information contained herein is in every respect accurate or complete.
Online PG Nepal QAE Kartik 5

ONLINE PG NEPAL
CEE MD/MS BASED MODEL TEST
(Wednesday, Kartik 5, 2077)
www.onlinepgnepal.com
Visit our website for Online Model Tests and various PG preparation related stuffs and activities.

Time: 3 Hours Full marks: 200


No negative markings

How to get Daily Viber Update services: Step wise


• Save our number 9801114177 in your Mobile contact list as Online PG Nepal (Saving number is important).
• Send us "Hello, my name is ……" message in Viber number of 9801114177. Send your Email address, MBBS college.
• We will send you updates about various MD/MS entrance related activities, notices, questions/Ans: s through Viber.
Any correction or explanation updates will be sent in Viber.

www.onlinepgnepal.com Page 2
Online PG Nepal QAE Kartik 5

Medicine
1. Which of the following malignancy is associated with Stauffers syndrome?
a. Hepatocellular carcinoma
b. Renal cell carcinoma
c. Squamous cell carcinoma lung
d. Hodgkin's lymphoma

Ans: b
Ref: Harrison’s Principles of Internal Medicine; 20th Edition, Page No: 617
A spectrum of paraneoplastic syndromes has been associated with RCC, including erythrocytosis, hypercalcemia,
nonmetastatic hepatic dysfunction (Stauffer’s syndrome), and acquired dysfibrinogenemia.
The term Stauffer’s syndrome has been used for intrahepatic cholestasis specifically associated with renal cell cancer.

2. Which of the following drug is used to differentiate 2nd order and 3rd order neuron lesion in a case of Horner
syndrome?
a. Apraclonidine
b. Cocaine
c. Pilocarpine
d. Hydroxyamphetamine

Ans: d
Ref: SachinKedar, MD et al., Horner syndrome
 Localization of the lesion — Hydroxyamphetamine eye drops will differentiate between a lesion affecting the first-
(brainstem or cervical cord) or second-order (chest or neck) neuron and one affecting the third-order or
postganglionic neuron (above the superior cervical ganglion at the carotid bifurcation). There is no pharmacologic
test to distinguish between first- and second-order lesions. Because cocaine may interfere with the uptake and
efficacy of hydroxyamphetamine drops, it is recommended that 24 to 72 hours elapse between the two tests.
 Hydroxyamphetamine releases stored norepinephrine from the postganglionic adrenergic nerve endings. One
hour after instillation of 1 percent hydroxyamphetamine, a normal pupil and a first- or second-order Horner pupil
will dilate, whereas a third-order Horner pupil will not dilate as well as the normal pupil. The test is positive for
postganglionic Horner lesions when the anisocoria increases by at least 1 mm. This test has a sensitivity of 93 to 96
percent and a specificity of 84 percent for detecting postganglionic lesions.

3. Anisocoria more in light than in dark is due to:


a. Adie's tonic pupil
b. Physiological anisocoria
c. 3rd nerve palsy
d. Horner syndrome

Ans: c
Ref: SachinKedar, MD et al., Approach to anisocoria

www.onlinepgnepal.com Page 3
Online PG Nepal QAE Kartik 5

4. The key feature that differentiates thunderclap headache from other heheadache is:
a. Rapidity with which it develops
b. Severity
c. Headache that is associated with meningismus
d. Headache that is associated with loss of consciousness or cranial nerve deficit

Ans: a
Ref: Todd J Schwedt, MD et al.
Overview of thunderclap headache

www.onlinepgnepal.com Page 4
Online PG Nepal QAE Kartik 5

 Thunderclap headache (TCH) refers to a severe headache of sudden onset. Its explosive and unexpected nature is
likened to a "clap of thunder."
 A TCH is a very severe headache of abrupt onset that reaches its maximum intensity within one minute or less of
onset. The key feature that differentiates TCH from other headaches is the rapidity with which it develops;
extreme severity alone is insufficient.
 Patients may present with TCH either in isolation or accompanied by additional symptoms and signs (eg,
meningismus, fever, tinnitus, orthostatic worsening of headache, altered mental state, seizure, motor or sensory
deficits, or cranial nerve palsies) that reflect the underlying cause.

5. Which among the following is dorsal midbrain syndrome?


a. Benedikt syndrome
b. Weber syndrome
c. Parinaud syndrome
d. Claude syndrome

Ans: c
Ref: Andrew G Lee et al., Supranuclear disorders of gaze
The constellation of neuroophthalmologic findings seen with pretectal lesions (including abnormalities of vertical gaze and
convergence) has been variously called Parinaud syndrome, the Sylvian aqueduct syndrome, the pretectal syndrome, the
dorsal midbrain syndrome, and the Koerber-Salus-Elschnig syndrome.
Rest all are ventral midbrain syndromes.
 Contralateral ataxia will be present if the red nucleus/superior cerebellar peduncle is involved
 Cerebellar tremor may be present in Claude syndrome (ipsilateral third nerve palsy and contralateral cerebellar
signs)
 Contralateral hemiparesis may be present in a cerebral peduncle lesion (Weber syndrome)
 Contralateral choreiform movements or tremor are present when red nucleus/substantia nigra involvement
occurs (Benedikt syndrome)

6. All of the following are pyrogenic cytokines except?


a. IL-1
b. IL-4
c. IL-6
d. TNF

Ans: b
Ref: Harrison’s Principles of Internal Medicine; 20th Edition, Page No: 103
 Cytokines are small proteins (molecular mass, 10,000-20,000 Da) that regulate immune, inflammatory, and
hematopoietic processes. For example, the elevated leukocytosis seen in several infections with an absolute
neutrophilia is attributable to the cytokines interleukin (IL) 1 and IL-6.
 Some cytokines also cause fever; formerly referred to as endogenous pyrogens, they are now called pyrogenic
cytokines. The pyrogenic cytokines include IL-1, IL-6, tumor necrosis factor (TNF), and ciliary neurotropic factor, a
member of the IL-6 family.
 Fever is a prominent side effect of interferon α therapy. Each pyrogenic cytokine is encoded by a separate gene,
and each has been shown to cause fever in laboratory animals and in humans.

www.onlinepgnepal.com Page 5
Online PG Nepal QAE Kartik 5

 When injected into humans at low doses (10-100 ng/kg), IL-1 and TNF produce fever; in contrast, for IL-6, a dose of
1-10 μg/kg is required for fever production.

7. Which of the following is not the component of Gerstmann’s Syndrome?


a. Aphasia
b. Acalculia
c. Dysgraphia
d. Finger anomia

Ans: a
Ref: Harrison’s Principles of Internal Medicine; 20th Edition, Page No: 160
Gerstmann’s Syndrome:
 The combination of acalculia (impairment of simple arithmetic), dysgraphia (impaired writing), finger anomia (an
inability to name individual fingers such as the index and thumb), and right-left confusion (an inability to tell
whether a hand, foot, or arm of the patient or examiner is on the right or left side of the body) is known as
Gerstmann’s syndrome.
 In making this diagnosis, it is important to establish that the finger and left-right naming deficits are not part of a
more generalized anomia and that the patient is not otherwise aphasic.
 When Gerstmann’s syndrome arises acutely and in isolation, it is commonly associated with damage to the
inferior parietal lobule (especially the angular gyrus) in the left hemisphere.

8. Which of the following is impaired in conduction aphasia?


a. Speech output
b. Reading comprehension
c. Reading aloud
d. Memory

Ans: c
Ref: Harrison’s Principles of Internal Medicine; 20th Edition, Page No: 159
Conduction Aphasia:
 Speech output is fluent but contains many phonemic paraphasias, comprehension of spoken language is intact,
and repetition is severely impaired. Naming elicits phonemic paraphasias, and spelling is impaired. Reading aloud
is impaired, but reading comprehension is preserved.
 The responsible lesion, usually a CVA in the temporoparietal or dorsal perisylvian region, interferes with the
function of the phonological loop interconnecting Broca’s area with Wernicke’s area. Occasionally, a transient
Wernicke’s aphasia may rapidly resolve into a conduction aphasia.
 The paraphasic output in conduction aphasia interferes with the ability to express meaning, but this deficit is not
nearly as severe as the one displayed by patients with Wernicke’s aphasia. Associated neurologic signs in
conduction aphasia vary according to the primary lesion site.

9. Larger “red cell distribution width” suggests:


a. Thalassemia
b. Megaloblastic anemia
c. Hereditary spherocytosis
d. Iron deficiency anemia

www.onlinepgnepal.com Page 6
Online PG Nepal QAE Kartik 5

Ans: d
Ref: Harrison’s Principles of Internal Medicine; 20th Edition, Page No: 379
 The electronic cell counter provides an independent assessment of variability in red cell size. It measures the
range of red cell volumes and reports the results as “red cell distribution width” (RDW). This value is calculated
from the MCV; thus, cell width is not being measured but cell volume is.
 The term is derived from the curve displaying the frequency of cells at each volume, also called the distribution.
The width of red cell volume distribution curve is what determines the RDW.
 The RDW is calculated as follows: RDW = (standard deviation of MCV ÷ mean MCV) × 100.
 In the presence of morphologic anisocytosis, RDW (normally 11-14%) increases to 15-18%.
 The RDW is useful in at least two clinical settings. In patients with microcytic anemia, the differential diagnosis is
generally between iron deficiency and thalassemia.
 In thalassemia, the small red cells are generally of uniform size with a normal small RDW. In iron deficiency, the
size variability and the RDW are large.
 In addition, a large RDW can suggest a dimorphic anemia when a chronic atrophic gastritis can produce both
vitamin B12 malabsorption to produce macrocytic anemia and blood loss to produce iron deficiency. In such
settings, RDW is also large.

10. Generalized lymphadenopathy is defined as involvement of:


a. 2 or more contiguous lymph nodes
b. 3 or more contiguous lymph nodes
c. 2 or more non-contiguous lymph nodes
d. 3 or more non-contiguous lymph nodes

Ans: d
Ref: Harrison’s Principles of Internal Medicine; 20th Edition, Page No: 412
 The physical examination can provide useful clues such as the extent of lymphadenopathy (localized or
generalized), size of nodes, texture, presence or absence of nodal tenderness, signs of inflammation over the
node, skin lesions, and splenomegaly. A thorough ear, nose, and throat (ENT) examination is indicated in adult
patients with cervical adenopathy and a history of tobacco use.
 Localized or regional adenopathy implies involvement of a single anatomic area. Generalized adenopathy has
been defined as involvement of three or more noncontiguous lymph node areas.
 Many of the causes of lymphadenopathy can produce localized or generalized adenopathy, so this distinction is of
limited utility in the differential diagnosis.
 Nevertheless, generalized lymphadenopathy is frequently associated with nonmalignant disorders such as
infectious mononucleosis (Epstein-Barr virus [EBV] or cytomegalovirus [CMV]), toxoplasmosis, AIDS, other viral
infections, systemic lupus erythematosus (SLE), and mixed connective tissue disease.
 Acute and chronic lymphocytic leukemias and malignant lymphomas also produce generalized adenopathy in
adults.

11. Drug implicated in nail discoloration is:


a. Indinavir
b. Zidovudine
c. Lithium
d. Retinoids

www.onlinepgnepal.com Page 7
Online PG Nepal QAE Kartik 5

Ans: b
Ref: Harrison’s Principles of Internal Medicine; 20th Edition, Page No: 365
ONYCHOLYSIS:
Onycholysis occurs with tetracyclines, fluoroquinolones, retinoids, NSAIDs, and others, including many chemotherapeutic
agents, and may be triggered by exposure to sunlight.

ONYCHOMADESIS:
Onychomadesis is caused by temporary arrest of nail matrix mitotic activity. Common drugs reported to induce
onychomadesis include carbamazepine, lithium, retinoids, and chemotherapeutic agents.

PARONYCHIA:
Paronychia and multiple pyogenic granuloma with progressive and painful periungual abscess of fingers and toes are side
effects of systemic retinoids, lamivudine, indinavir, and anti-EGFR monoclonal antibodies.

NAIL DISCOLORATION:
Some drugs—including anthracyclines, taxanes, fluorouracil, psoralens, and zidovudine—may induce nail bed
hyperpigmentation through melanocyte stimulation. It appears to be reversible and dose dependent.

12. Michaelis-Gutmann bodies are seen in:


a. Renal malacoplakia
b. Xanthogranulomatous Pyelonephritis
c. Renal abscess
d. Emphysematous Pyelonephritis

Ans: a
Ref: Comprehensive nephrology,6th edition, Page 635
Renal Malacoplakia:
 Malacoplakia is a chronic granulomatous disorder of unknown etiology involving the genitourinary,
gastrointestinal, skin, and pulmonary systems.
 It is characterized by an unusual inflammatory reaction to a variety of infections and is manifested by the
accumulation of macrophages containing calcified bacterial debris called Michaelis-Gutmann bodies.
 The underlying disorder appears to be a monocyte-macrophage bactericidal defect.
 The diagnosis is made by histologic examination of involved tissue. Genitourinary malacoplakia, most often
involving the bladder, is usually associated with gram-negative UTI.
 Patients with renal malacoplakia generally have fever, flank pain, pyuria and hematuria, bacteriuria, and, if both
kidneys are involved, impaired renal function.
 CT usually shows enlarged kidneys with areas of poor enhancement, and the condition may be indistinguishable
from other infectious or neoplastic lesions.
 On occasion, the malacoplakia may extend through the renal capsule into the perinephric space, simulating a
renal carcinoma.
 Treatment consists of therapy with a broad-spectrum antimicrobial, attempted correction of any underlying
complicating conditions, and improvement of renal function. Nephrectomy is recommended for advanced
unilateral disease.

www.onlinepgnepal.com Page 8
Online PG Nepal QAE Kartik 5

13. The effect of calcium supplementation on Blood pressure is:


a. It decreases BP
b. It increases BP
c. No effect on BP
d. Variable effect (Depends on baseline BP)

Ans: a
Ref: Comprehensive nephrology,6th edition, Page 427
Cross-sectional population surveys of self-reported nutrient intake suggest an inverse relationship between calcium
intake and BP. The relationship is more convincing at low levels of calcium consumption (<300 to 600 mg/day). There
may be a threshold of 700 to 800 mg/day, above which any further reduction in BP is attenuated. A meta-analysis of
randomized calcium supplementation trials (mostly with 1 or 1.5 g calcium daily) demonstrated reductions in SBP (−0.9 to
−1.7 mm Hg) that are of liYle clinical importance.39 Although calcium in milk may contribute to BP lowering, dairy
products may lower BP by other mechanisms. Biologically active peptides formed during the milk fermentation
process, such as the casein-derived tripeptides isoleucine-proline-proline and valine-proline-proline, have ACE-
inhibiting properties.

14. The most common renal sequelae with the use of lithium is:
a. Nephrogenic diabetes insipidus
b. Chronic tubulointerstitial nephritis
c. FSGS
d. Renal cysts

Ans: a
Ref: Harrison’s Principles of Internal Medicine; 20th Edition, Page No: 2162
LITHIUM-ASSOCIATED NEPHROPATHY
 The use of lithium salts for the treatment of manic-depressive illness may have several renal sequelae, the most
common of which is nephrogenic diabetes insipidus manifesting as polyuria and polydipsia.
 Lithium accumulates in principal cells of the collecting duct by entering through the epithelial sodium channel
(ENaC), where it inhibits glycogen synthase kinase 3β and downregulates vasopressin-regulated aquaporin water
channels.
 Less frequently, chronic TIN develops after prolonged (>10-20 years) lithium use and is most likely to occur in
patients who have experienced repeated episodes of toxic lithium levels.

15. Sick sinus syndrome includes all of the following except:


a. Sinus bradycardia
b. Sinus tachycardia
c. Alternating bradycardia and atrial tachyarrhythmias
d. Sinus pauses

Ans: b
Ref: Uptodate, Sinus node dysfunction: Clinical manifestations, diagnosis and evaluation; Munther K Homoud, MD et al
The diagnosis of Sinus node dysfunction(aka Sick sinus syndrome) in persons with suggestive symptoms is often made
from the surface ECG. ECG manifestations can include:
 Periods of inappropriate, and often severe (less than 50 beats per minutes), bradycardia

www.onlinepgnepal.com Page 9
Online PG Nepal QAE Kartik 5

 Sinus pauses, arrest, and sinoatrial (SA) exit block with, and often without, appropriate atrial and junctional escape
rhythms. The failure of escape pacemakers may lead to symptoms including syncope
 Alternating bradycardia and atrial tachyarrhythmias in over 50 percent of cases.Atrial fibrillation is most common,
but atrial flutter and paroxysmal supraventricular tachycardias (ie, due to atrial tachycardia) may also occur.
 Atrial arrhythmias seem to develop slowly over time, possibly the result of a progressive pathological process that
affects the SA node and the atrium.

16. Rasmussen's aneurysm can rupture and cause hemoptysis. It arises from:
a. Bronchial artery
b. Bronchial vein
c. Pulmonary artery
d. Pulmonary vein

Ans: c
Ref: Harrison’s Principles of Internal Medicine; 20th Edition, Page No: 233
In patients with tuberculosis, development of cavitary disease is frequently the source of bleeding but rarer complications
such as the erosion of a pulmonary artery aneurysm into a preexisting cavity (i.e., Rasmussen’s aneurysm) can also be the
source.

17. ADA level to label pleural effusion to be tubercular should be above


a. 10 IU/L
b. 20 IU/L
c. 30 IU/L
d. 40 IU/L

Ans: d
Ref: Harrison’s Principles of Internal Medicine; 20th Edition, Page No: 2008
Patients with tuberculous pleuritis present with fever, weight loss, dyspnea, and/or pleuritic chest pain. The pleural fluid
is an exudate with predominantly small lymphocytes. The diagnosis is established by demonstrating high levels of TB
markers in the pleural fluid (adenosine deaminase >40 IU/L or interferon γ >140 pg/mL). Alternatively, the diagnosis can
be established by culture of the pleural fluid, needle biopsy of the pleura, or thoracoscopy.

18. Deep Brain Stimulation done for Parkinson’s disease primarily targets:
a. Globus Pallidus internus
b. Globus Pallidus externus
c. Substantia Nigra compacta
d. Substantia Nigra reticulata

Ans: a
Ref: Harrison’s Principles of Internal Medicine; 20th Edition, Page No: 3129
 DBS for PD primarily targets the STN or the GPi.
 It provides dramatic results, particularly with respect to tremor and reducing both “off” time and dyskinesias, but
does not provide superior clinical benefits or improve features that do not respond to levodopa such as freezing,
falling, and dementia.

www.onlinepgnepal.com Page 10
Online PG Nepal QAE Kartik 5

 The procedure is thus primarily indicated for patients who suffer disability resulting from severe tremor, or
levodopa-induced motor complications that cannot be satisfactorily controlled with drug manipulation. In such
patients, DBS has been shown to provide therapy.
 Recent studies indicate that benefits following DBS of the STN and GPi are comparable, but that GPi stimulation
may be associated with a reduced frequency of depression. Although not all PD patients are candidates, the
procedure can be profoundly beneficial for many.

Surgery
19. Cryoprecipitate is rich source of factor:
a. Factor V
b. Factor IV
c. Factor VIII
d. Factor X

Ans: c
Ref: Bailey and Love’s Short Practice of Surgery 27th Edition, Page No: 21
Cryoprecipitate is a supernatant precipitate of FFP and is rich in factor VIII and fibrinogen. It is stored at −30°C with a 2-
year shelf life. It is given in low fibrinogen states or factor VIII deficiency.
Platelets
• Platelets are supplied as a pooled platelet concentrate and contain about 250 × 109/L. Platelets are stored on a
special agitator at 20–24°C and have a shelf life of only 5 days. Platelet transfusions are given to patients with
thrombocytopenia or with platelet dysfunction who are bleeding or undergoing surgery.
• Patients are increasingly presenting on antiplatelet therapy such as aspirin or clopidogrel for reduction of
cardiovascular risk.
• Aspirin therapy rarely poses a problem but control of hemorrhage on the more potent platelet inhibitors can be
extremely difficult. Patients on clopidogrel who are actively bleeding and undergoing major surgery may require
almost continuous infusion of platelets during the course of the procedure.
• Arginine vasopressin or its analogues (DDAVP) have also been used in this patient group, although with limited
success.
Prothrombin complex concentrates
• Prothrombin complex concentrates (PCC) are highly purified concentrates prepared from pooled plasma. They
contain factors II, IX and X. Factor VII may be included or produced separately. It is indicated for the emergency
reversal of anticoagulant (warfarin) therapy in uncontrolled hemorrhage.

20. Advantages of minimal access surgery are all except:


a. Decreased heat loss
b. Improved visualization
c. Decrease mobility
d. Decrease in wound pain

Ans: c
Ref: Bailey and Love’s Short Practice of Surgery 27th Edition, Page No: 407
Advantages of minimal access surgery
• Decrease in wound size

www.onlinepgnepal.com Page 11
Online PG Nepal QAE Kartik 5

• Reduction in wound infection, dehiscence, bleeding, herniation and nerve entrapment


• Decrease in wound pain
• Improved mobility
• Decreased wound trauma
• Decreased heat loss
• Improved visualization

21. In which of the following type of carcinoma breast, comedo growth pattern is seen:
a. Lobular carcinoma insitu
b. Ductal carcinoma insitu
c. Infiltrating lobular carcinoma
d. Invasive ductal

Ans: b
Ref: SRB's Manual of Surgery; 5th Edition, Page No: 531
Noninvasive epithelial
 LCIS
 DCIS (intraductal)—papillary, solid, cribriform, comedo
Invasive epithelial
 Invasive lobular—10%
 Invasive ductal
o Invasive ductal with NST (no special type)/NOS (not otherwise specified)—70%
o Tubular—2%
o Colloid—2%
o Medullary—5%
o Medullary variant—basal like
o Invasive cribriform—2%
o Invasive papillary—1%
o Adenoid cystic—1%
o Metaplastic—1%
Mixed connective tissue and epithelial
Phyllodes, angiosarcoma, carcinosarcoma

22. Which of the following doesn’t take radioactive iodine?


a. Papillary carcinoma thyroid
b. Follicular carcinoma thyroid
c. Medullary carcinoma thyroid
d. Hurthle cell carcinoma thyroid

Ans: c
Ref: Bailey and Love’s Short Practice of Surgery 27th Edition, Page No: 820-821
Medullary carcinoma
• These are tumors of the parafollicular (C cells) derived from the neural crest. rather than the cells of the thyroid
follicle as are other primary thyroid carcinomas.

www.onlinepgnepal.com Page 12
Online PG Nepal QAE Kartik 5

• The cells are not unlike those of a carcinoid tumour and on histological analysis a characteristic amyloid stroma is
seen.
• High levels of serum calcitonin and carcinoembryonic antigen are produced by many medullary tumours, which
should be tested for in suspected cases.
• Calcitonin levels fall after resection and rise again with recurrence, making it a valuable marker in the follow-up of
patients with this disease.
• Diarrhoea is a feature in 30% of cases and this may be due to 5-hydroxytryptamine or prostaglandins produced by
the tumour cells.
• Some tumours are familial and account for 10–20% of all cases. Medullary carcinoma may occur in combination
with adrenal phaeochromocytoma and hyperparathyroidism (HPT) (usually due to hyperplasia) in the syndrome
known as multiple endocrine neoplasia type 2A (MEN-2A).
• The familial form of the disease frequently affects children and young adults, whereas the sporadic cases occur at
any age with no sex predominance.
• When the familial form is associated with prominent mucosal neuromas involving the lips, tongue and inner
aspect of the eyelids, with a Marfanoid habitus, the syndrome is referred to as MEN type 2B.
• Involvement of lymph nodes occurs in 50–60% of cases of medullary carcinoma and blood-borne metastases are
common. As would be expected, tumours are not TSH dependent and do not take up radioactive iodine.
• The prognosis is variable and depends on the stage at diagnosis.
• Any nodal involvement virtually eliminates the prospect of cure and, unfortunately, even small tumours confined
to the thyroid gland may have spread by the time of diagnosis, particularly in familial cancers. In common with
many endocrine tumours the progression of disease may be very slow, with a characteristically indolent course
and long survival, even in the absence of cure.
• In familial cases of medullary thyroid cancer, genetic screening of relatives should be recommended.
• This is a complex subject but individuals identified can be risk stratified dependent on the genetic abnormality.
This information can be used to make recommendations concerning prophylactic thyroidectomy.
• Some relatives may be monitored into adulthood with serial calcitonin monitoring. In contrast, the highest-risk
mutations are associated with early-onset disease and total thyroidectomy is recommended during infancy.

23. During surgical exploration for Hydatid cyst of the liver, any of the following agents can be used as scolicidal agent
except:
a. Hypertonic sodium chloride
b. Formalin
c. Cetrimide
d. Hydrogen peroxide

Ans: b
Ref: SRB's Manual of Surgery; 5th Edition, Page No: 605
Scolicidal agents
• Cetrimide—can cause acidosis
• Alcohol 80%—can cause cholangitis
• Hypertonic saline—hypernatraemia
• Sodium hypochlorite—hypernatraemia
• Hydrogen peroxide
In cases with biliary communication only hypertonic saline (15–20%) is used (not other agents).

www.onlinepgnepal.com Page 13
Online PG Nepal QAE Kartik 5

24. Which of the following is not component of APACHE Score?


a. Serum potassium
b. Serum sodium
c. Serum calcium
d. Serum creatinine

Ans: c
Ref: SRB's Manual of Surgery; 5th Edition, Page No: 688
Acute Physiology and Chronic Health Evaluation (APACHE II) score >8 points predicts 11 to 18% mortality.
Equation includes the following factors:s: age, rectal temperature, mean arterial pressure, heart rate, PaO2, arterial pH,
serum potassium, serum sodium, serum creatinine
creatinine,, hematocrit, white blood cell count, Glasgow coma scale score.

25. Indications of surgery in benign prostatic hyperplasia are all except:


a. Hemorrhage
b. Chronic retention with renal impairment
c. Enlarged prostate
d. Acute retention

Ans: c
Ref: Bailey and Love’s Short Practice of Surgery 27th Edition, Page No: 1463
MANAGEMENT OF MEN WITH BENIGN PROSTATIC HYPERPLASIA OR BLADDER OUT OUTFLOW
FLOW OBSTRUCTION
Strong indications for treatment (usually prostatectomy) include:
• Acute retention in fit men with no other cause for retention (drugs, constipation, recent operation, etc.) (accounts
for 25% of prostatectomies).
• Chronic retention and renal impairment
impairment:: a residual urine of 200mL or more, a raised blood urea, hydroureter or
hydronephrosis demonstrated on urography and uraemic manifestations (accounts for 15% of prostatectomies).
• Complications of bladder outflow obstruction
obstruction: stone, infection and
d diverticulum formation.
• Haemorrhage:: occasionally, venous bleeding from a ruptured vein overlying the prostate will require
prostatectomy to be performed.
• Elective prostatectomy for severe symptoms: this accounts for about 60% of prostatectomies. Increasing Increas
difficulty in micturition, with considerable frequency day and night, delay in starting and a poor stream are the

www.onlinepgnepal.com Page 14
Online PG Nepal QAE Kartik 5

usual symptoms for which prostatectomy is advised. Frequency alone is not a strong indication for prostatectomy.
The natural progression off outflow obstruction is variable and rarely gets worse after 10 years. Severe symptoms,
a low maximum flow rate (<10mL/s) and an increased residual volume of urine (100 (100–250mL)
250mL) are relatively strong
indications for operative treatment. The exact cut
cut-off for operative or non-operative
operative treatment will depend on
careful discussion between the patient and the urologist. Holmium laser enucleation of the prostate (HOLEP) or
open prostatectomy should be considered for very large prostates.

26. Unilateral claudication


ion in the thigh and calf and sometimes the buttock indicates:
a. Aorta-iliac obstruction
b. Iliac obstruction
c. Femoropopliteal obstruction
d. Distal obstruction

Ans: b
Ref: Bailey and Love’s Short Practice of Surgery 27th Edition, Page No: 944
Table: Relationship of clinical findings to site of disease

27. The most common clinical pattern of basal cell carcinoma is:
a. Morphea form
b. Keratotic
c. Superficial
d. Nodular

Ans: d
Ref: Bailey and Love’s Short Practice of Surgery 27th Edition, Page No: 604
604-605
Basal cell carcinoma

www.onlinepgnepal.com Page 15
Online PG Nepal QAE Kartik 5

• This is usually a slow-growing, locally-invasive, malignant tumour of pluripotential epithelial cells arising from
basal epidermis and hair follicles; hence, it affects the pilosebaceous skin.
EPIDEMIOLOGY
• The strongest predisposing factor to BCC is UVR.
• It occurs in the elderly or the middle-aged after excessive sun exposure, with 95% occurring between the ages of
40 and 80 years.
• The incidence of BCC rises with proximity to the equator, although 33% arise in parts of the body not usually sun
exposed. Other predisposing factors include exposure to arsenical compounds, coal tar, aromatic hydrocarbons,
ionising radiation and genetic skin cancer syndromes. White-skinned people are almost exclusively affected. BCC is
more common in men than women.
PATHOGENESIS
• BCCs have no apparent precursor lesions and their development is proportional to the initial dose of the
carcinogen, but not duration of exposure.
• The most likely model of pathogenesis for BCCs involves mesodermal factors as intrinsic promoters coupled with
an initiation step.
• BCCs metastasize extremely rarely.
MACROSCOPIC
• BCC can be divided into localised (nodular; nodulocystic; cystic; pigmented and naevoid) and generalised
(superficial: multifocal and superficial spreading; or infiltrative: morphoeic, ice pick and cicatrizing).
• Nodular and nodulocystic variants account for 90% of BCC.
MICROSCOPIC
• Twenty-six histological subtypes have been described. The characteristic finding is of ovoid cells in nests with a
single ‘palisading’ layer.
• It is only the outer layer of cells that actively divide, explaining why tumour growth rates are slower than their cell
cycle speed would suggest, and why incompletely excised lesions are more aggressive.
• Morphoeic BCCs synthesize type 4 collagenase and so spread rapidly.

28. A patient with head injury opens eyes to painful stimulus, uses inappropriate words and localizes pain. what is GCS
score?
a. 8
b. 9
c. 10
d. 11

Ans: c
Ref: Bailey and Love’s Short Practice of Surgery 27th Edition, Page No: 331

www.onlinepgnepal.com Page 16
Online PG Nepal QAE Kartik 5

Table: Glasgow Coma Scale score for head injury

Opens eyes to painful stimulus = 2


Inappropriate words = 3
Localizes to pain = 5
So, total score is 2 + 3 + 5 = 10.

29. Indication of PCNL are all EXCEPT:


a. Stone size >2cm
b. Hounsfield unit >1000
c. Multiple stone
d. Coagulopathy

Ans: d
Ref: Sabiston Surgery 20th Edition, Page No: 2084
 Complications of PCNL are most significant because of the more invasive nature of the procedure; these include
sepsis, renal hemorrhage, renal collecting system injury, and damage to adjacent organs and an viscera. PCNL may
result in hydrothorax or pneumothorax from transpleural or peripleural access tracks that require evacuation.
 For larger renal stones or select proximal ureteral stones, PCNL is preferred because of the larger working
endoscopes and better ter instrumentation for stone fragmentation. The advantage of PCNL is that numerous
intracorporeal lithotripsy devices are available, and large stones can be rapidly fragmented.

30. Investigation of choice for the diagnosis of GERD is:


a. Endoscopy
b. Manometry
c. 24 hour pH monitoring
d. Barium swallow

Ans: c
Ref: Bailey and Love’s Short Practice of Surgery 27th Edition, Page No: 1071

www.onlinepgnepal.com Page 17
Online PG Nepal QAE Kartik 5

 Prolonged measurement of pH is now accepted as the most accurate method for the diagnosis of gastro-
oesophageal reflux. It is particularly useful in patients with atypical reflux symptoms, those without endoscopic
oesophagitis and when patients respond poorly to intensive medical therapy.
 A small pH probe is passed into the distal oesophagus and positioned 5 cm above the upper margin of the LOS, as
defined by manometry. The probe is connected to a miniature digital recorder that is worn on a belt and allows
most normal activities. Patients mark symptomatic events such as heartburn. A 24-hour recording period is usual,
and the pH record is analysed by an automated computer program.
 An oesophageal pH <4 at the level of the pH electrode is conventionally considered the cut-off value and, in most
oesophageal laboratories, the total time when pH is <4 in a 24-hour period does not exceed 4% in a healthy adult.
Most laboratories use a scoring system (Johnson–DeMeester) to create a numerical value, above which reflux is
considered pathological.

31. In which type of carcinoma breast, would you consider biopsy of opposite breast?
a. Adenocarcinoma poorly differentiated
b. Medullary carcinoma
c. Lobular carcinoma
d. Comedo carcinoma

Ans: c
Ref: Bailey and Love’s Short Practice of Surgery 27th Edition, Page No: 872 | Ref: Schwartz’s Principles of Surgery, 10th
Edition, Page No: 520
Invasive lobular carcinoma is commonly multifocal and/or bilateral. Invasive cancer may develop in either breast,
regardless of which breast harbored the initial focus of lobular carcinoma is situ (LCIS), and is detected synchronously with
LCIS in 5% of cases. For these reasons, LCIS is regarded as a marker of increased risk for invasive breast cancer rather than
as an anatomic precursor.

32. Most common site of cholangiocarcinoma is


a. Distal biliary duct
b. Hilum
c. Intrahepatic duct
d. Multifocal

Ans: b
Ref: Sabiston Surgery 20th Edition, Page No: 1514
Historically, evaluation and management of cholangiocarcinoma required arbitrary division of the bile duct into thirds
based on the location of obstruction. Lesions of the middle third, however, are decidedly rare, so investigations have
recently focused on perihilar and intrahepatic lesions, known as proximal lesions, versus those involving the periampullary
region, known as distal disease. More than two thirds of all cholangiocarcinomas involve the proximal biliary tree near the
bifurcation, known as Klatskin tumor.

33. The following statements are true about Peyronie’s disease except:
a. Patient presents with complaints of painful erection
b. Condition affects adolescent males
c. The condition can be associated with Dupuytren’s contracture of the tendon of the hand
d. Spontaneous regression occurs in 50% of the cases

www.onlinepgnepal.com Page 18
Online PG Nepal QAE Kartik 5

Ans: b
Ref: Bailey and Love’s Short Practice of Surgery 26th Edition, Page No: 1372
Peyronie’s Disease
• Peyronie’s disease (plastic induration of penis/ penile fibromatosis) usually seen over 40 years of age.
• It is due to fibrous plaques in one or both corpus cavernosum of varying sizes involving tunica albuginea which
may later calcify or ossify.
• Cause remains obscure; the dense fibrous plaque is microscopically consistent with findings of severe vasculitis.
• Palmar fibromatosis (Dupuytren’s contracture), plantar fibromatosis and penile fibromatosis (Peyronie’s disease)
are components of the same pathological process called superficial fibromatosis.
Clinical Features
• Painful erection, curvature of penis and poor erection distal to involved area.
• No pain when the penis is in non erect state.
• Palpable induration or mass appears usually on the dorsolateral aspect of the penis.
Treatment
• Spontaneous remission occurs in about 50% cases, so observation and emotional support advised initially.
• If the penile deformity is distressing, Nesbitt’s operation can be performed to straighten the penis.
• Nesbitt operation: Straightening of penis by placing non-absorbable sutures in corpus cavernosum opposite to the
plaque.

34. The best guide for measure for organ perfusion:


a. BP
b. Urine output
c. CVP
d. Oxygen saturation

Ans: b
Ref: Bailey and Love’s Short Practice of Surgery 27th Edition, Page No: 16-17
Central venous pressure
There is no ‘normal’ central venous pressure (CVP) for a shocked patient, and reliance cannot be placed on an individual
pressure measurement to assess volume status. Some patients may require a CVP of 5 cmH2O, whereas some may require
a CVP of 15 cmH2O or higher. Further, ventricular compliance can change from minute to minute in the shocked state, and
CVP is a poor reflection of end diastolic volume (preload).

The best measure of organ perfusion and the best monitor of the adequacy of shock therapy remains the urine output.
However, this is an hourly measure and does not give a minute-to-minute view of the shocked state.

35. Preferred route for giving TPN for a patient less than 14 days and there is no other use of central vein:
a. IJV
b. EJV
c. Peripheral vein
d. Subclavian vein

Ans: c
Ref: Bailey and Love’s Short Practice of Surgery 27th Edition, Page No: 287

www.onlinepgnepal.com Page 19
Online PG Nepal QAE Kartik 5

Peripheral
Peripheral feeding is appropriate for short-term feeding of up to 2 weeks. Access can be achieved either by means of a
dedicated catheter inserted into a peripheral vein and maneuvered into the central venous system (peripherally
inserted central venous catheter (PICC) line) or by using a conventional short cannula in the wrist veins. The former
method has the advantage of minimizing inconvenience to the patient and clinician. PICC lines have a mean duration of
survival of 7 days.

Central
When the central venous route is chosen, the catheter can be inserted via the subclavian or internal or external jugular
vein. There is good evidence to show that the safest means of establishing central venous access is by insertion of lines
under ultrasound guidance; however, this will not be practicable for all cases. Most intensive care physicians and
anesthetists favor cannulation of internal or external jugular veins as these vessels are easily accessible. They suffer the
disadvantage that the exit site is situated inconveniently on the side of the neck, where repeated movements result in
disruption of the dressing with the attendant risk of sepsis. The infraclavicular subclavian approach is more suitable for
feeding as the catheter then lies flat on the chest wall, which optimizes nursing care.

36. Shoulder pain in patients after laparoscopy is managed by:


a. USG of right shoulder
b. Arthroscopy of right shoulder
c. Intraarticular lignocaine injection
d. Oral paracetamol for 2-3 days

Ans: d
Ref: Bailey and Love’s Short Practice of Surgery 27th Edition, Page No: 115
Shoulder tip pain
The patient should be warned about this preoperatively and told that the pain is referred from the diaphragm and not due
to a local problem in the shoulders. It can be at its worst 24 hours after the operation. It usually settles within 2–3 days
and is relieved by simple analgesics, such as paracetamol.

Pediatrics
37. Object Permanence milestone develops at:
a. 6 month
b. 9 month
c. 12 month
d. 15 month

Ans: b
Ref: Nelson Textbook of Pediatrics, Elsevier, 20th Edition, Page No: 69
A major milestone is the achievement by 9 month of object permanence (constancy), the understanding that objects
continue to exist, even when not seen. At 4-7 mo of age, infants look down for a yarn ball that has been dropped but
quickly give up if it is not seen. With object constancy, infants persist in searching. They will find objects hidden under a
cloth or behind the examiner’s back. Peek-a-boo brings unlimited pleasure as the child magically brings back the other
player. Events seem to occur as a result of the child’s own activities.

www.onlinepgnepal.com Page 20
Online PG Nepal QAE Kartik 5

38. All of the following are features of Rett’s syndrome except:


a. Microcephaly
b. Regression of milestones
c. Focal convulsions
d. Cardiac arrhythmia

Ans: c
Ref: Nelson Textbook of Pediatrics, Elsevier, 20th Edition, Page No: 2916-297
Rett syndrome
• This syndrome is not strictly speaking a degenerative disease, but a disorder of early brain development marked
by a period of developmental regression and deceleration of brain growth after a relatively normal neonatal
course.
• It occurs predominantly in girls. The frequency is approximately 1 in 15,000-22,000 children.
• Rett syndrome is caused by mutations in MeCP2, a transcription factor that binds to methylated CpG islands and
silences transcription.
• Development may proceed normally until 1 yr of age, when regression of language and motor milestones and
acquired microcephaly become apparent.
• An ataxic gait or fine tremor of hand movements is an early neurologic finding. Most children develop peculiar
sighing respirations with intermittent periods of apnea that may be associated with cyanosis.
• The hallmark of Rett syndrome is repetitive hand-wringing movements and a loss of purposeful and spontaneous
use of the hands; these features may not appear until 2-3 yr of age.
• Autistic behavior is a typical finding in all patients.
• Generalized tonic-clonic convulsions occur in the majority and are usually well controlled by anticonvulsants.
Feeding disorders and poor weight gain are common.
• After the initial period of neurologic regression, the disease process appears to plateau, with persistence of the
autistic behavior.
• Cardiac arrhythmias may result in sudden, unexpected death at a rate that is higher than the general population.
Generally girls survive into adulthood.

39. All are features of Vitamin D intoxication except:


a. Nausea and vomiting
b. Muscular weakness
c. Metastatic calcification
d. Oliguria

Ans: d
Ref: OP Ghai Pediatrics 8th Edition, Page No: 113
Hypervitaminosis D
• An epidemic of 'idiopathic hypercalcemia' in infants, with anorexia, vomiting, hypertension, renal insufficiency and
failure to thrive in England in the 1950s was traced to an intake of vitamin D between 2,000 and 3,000 IU / day. In
adults, dosages of 10,000 IU / day of vitamin D for several months have resulted in marked disturbances in calcium
metabolism with hypercalcemia, hyperphosphatemia, hypertension, anorexia, nausea, vomiting, weakness,
polyuria, polydipsia, azotemia, nephrolithiasis, ectopic calcification and renal failure.

40. True about Tonic neck reflex is:

www.onlinepgnepal.com Page 21
Online PG Nepal QAE Kartik 5

a. Extension of arm on contralateral side, flexion on ipsilateral side


b. Extension of arm on ipsilateral side and, flexion on contralateral side
c. Extension of arms on both sides
d. Flexion of arms on both sides

Ans: b
Ref: Nelson Textbook of Pediatrics, Elsevier, 20th Edition, Page No: 2798
Primitive Reflexes
• Primitive reflexes appear and disappear at specific times during development, and their absence or persistence
beyond those times signifies CNS dysfunction. Although many primitive reflexes have been described, the Moro,
grasp, tonic neck, and parachute reflexes are the most clinically relevant.
• The Moro reflex is elicited by supporting the infant in a semi-erect position and then allowing the infant’s head to
fall backwards onto the examiner’s hand. A normal response consists of symmetric extension and abduction of the
fingers and upper extremities, followed by flexion of the upper extremities and an audible cry. An asymmetric
response can signify a fractured clavicle, brachial plexus injury, or hemiparesis. Absence of the Moro reflex in a
term newborn is ominous, suggesting significant dysfunction of the CNS.
• The grasp response is elicited by placing a finger in the open palm of each hand; by 37 wk of gestation, the reflex is
strong enough that the examiner can lift the infant from the bed with gentle traction.
• The tonic neck reflex is produced by manually rotating the infant’s head to 1 side and observing for the
characteristic fencing posture (extension of the arm on the side to which the face is rotated and flexion of the
contralateral arm). An obligatory tonic neck response, in which the infant becomes “stuck” in the fencing posture,
is always abnormal and implies a CNS disorder.
• The parachute reflex, which occurs in slightly older infants, can be evoked by holding the infant’s trunk and then
suddenly lowering the infant as if he or she were falling. The arms will spontaneously extend to break the infant’s
fall, making this reflex a prerequisite to walking.

41. A blue new born presents with cyanosis. The X-ray chest reveals oligemic lung field and normal sized heart. Most
likely diagnosis is:
a. Ebstein’s anomaly
b. TOF
c. Tricuspid atresia
d. TGA

Ans: c
Ref: Nelson Textbook of Pediatrics, Elsevier, 20th Edition, Page No: 2218
• Ebstein anomaly-There is cardiomegaly in Ebstein anomaly
• TGA-There is cardiomegaly and plethoric lung field seen in it
• In both TOF and tricuspid atresia there is oligemic lung fields and normal size heart but in TOF cyanosis is usually
not present at birth.

Clinical manifestations of Tricuspid atresia


• Some degree of cyanosis is usually evident at birth, with the extent depending on the degree of limitation to
pulmonary blood flow.
• An increased left ventricular impulse may be noted, in contrast to most other causes of cyanotic heart disease, in
which an increased right ventricular impulse is usually present.

www.onlinepgnepal.com Page 22
Online PG Nepal QAE Kartik 5

• The majority of patients have holosystolic murmurs audible along the left sternal border; the 2nd heart sound is
usually single.
• Pulses in the lower extremities may be weak or absent in the presence of transposition with coarctation of the
aorta. Patients with tricuspid atresia are at risk for spontaneous narrowing or even closure of the VSD, which can
occasionally occur rapidly and lead to a marked increase in cyanosis.

42. A child presents with fever since 24 hrs. History reveals 3 episodes of chest infections and passage of bulky foul
smelling stools. The most probable diagnosis is:
a. Maple syrup urine disease
b. Criggler Najar syndrome
c. Cystic fibrosis
d. Bilirubin conjugation defect

Ans: c
Ref: Nelson Textbook of Pediatrics, Elsevier, 20th Edition, Page No: 2103-2104
CLINICAL MANIFESTATIONS OF CYSTIC FIBROSIS
• Mutational heterogeneity and environmental factors appear responsible for highly variable involvement of the
lungs, pancreas, and other organs.
• A list of presenting manifestations is lengthy, although pulmonary and gastrointestinal presentations
predominate.
• With inclusion of CF newborn screening panels, an increasing proportion of children are diagnosed before
symptoms appear.
Respiratory Tract
• Cough is the most constant symptom of pulmonary involvement.
• At first, the cough may be dry and hacking, but eventually it becomes loose and productive. In older patients, the
cough is most prominent upon arising in the morning or after activity. Expectorated mucus is usually purulent.
• Some patients remain asymptomatic for long periods or seem to have prolonged but intermittent acute
respiratory infections.
• Others acquire a chronic cough in the 1st few wk of life, or they have pneumonia repeatedly. Extensive
bronchiolitis accompanied by wheezing is a frequent symptom during the 1st few yr of life.
• As lung disease slowly progresses, exercise intolerance, shortness of breath, and failure to gain weight or grow are
noted. Exacerbations of lung symptoms, presumably owing to more active airways infection, often require
repeated hospitalizations for effective treatment.
• Cor pulmonale, respiratory failure, and death eventually supervene unless lung transplantation is accomplished.
Colonization with B. cepacia and other multidrug-resistant organisms may be associated with particularly rapid
pulmonary deterioration and death.
• The rate of progression of lung disease is the chief determinant of morbidity and mortality. The course of lung
disease is largely independent of genotype.
• Male gender and exocrine pancreatic sufficiency are also associated with a slower rate of pulmonary function
decline.
• Early physical findings include increased anteroposterior diameter of the chest, generalized hyperresonance,
scattered or localized coarse crackles, and digital clubbing. Expiratory wheezes may be heard, especially in young
children.
• Cyanosis is a late sign. Common pulmonary complications include atelectasis, hemoptysis, pneumothorax, and cor
pulmonale; these usually appear beyond the 1st decade of life.

www.onlinepgnepal.com Page 23
Online PG Nepal QAE Kartik 5

• Even though the paranasal sinuses are virtually always opacified radiographically, acute sinusitis is infrequent.
Nasal obstruction and rhinorrhea are common, caused by inflamed, swollen mucous membranes or, in some
cases, nasal polyposis. Nasal polyps are most troublesome between 5 and 20 yr of age.
Intestinal Tract
• In 10-15% of newborn infants with CF, the ileum is completely obstructed by meconium (meconium ileus).
• The frequency is greater (≈30%) among siblings born subsequent to a child with meconium ileus and is particularly
striking in monozygotic twins, reflecting a genetic contribution from 1 or more modifying genes. Abdominal
distention, emesis, and failure to pass meconium appear in the 1st 24-48 hr of life.
• Abdominal radiographs show dilated loops of bowel with air–fluid levels and, frequently, a collection of granular,
“ground-glass” material in the lower central abdomen.
• Rarely, meconium peritonitis results from intrauterine rupture of the bowel wall and can be detected
radiographically as the presence of peritoneal or scrotal calcifications.
• Meconium plug syndrome occurs with increased frequency in infants with CF but is less specific than meconium
ileus. Ileal obstruction with fecal material (distal intestinal obstruction syndrome) occurs in older patients, causing
cramping abdominal pain and abdominal distention.
• More than 85% of affected children show evidence of protein and fat malabsorption from exocrine pancreatic
insufficiency. Symptoms include frequent, bulky, greasy stools and failure to gain weight even when food intake
appears to be large.
• Characteristically, stools contain readily visible droplets of fat. A protuberant abdomen, decreased muscle mass,
poor growth, and delayed maturation are typical physical signs.
• Excessive flatus may be a problem.

Biliary Tract
• Manifestations can include icterus, ascites, hematemesis from esophageal varices, and evidence of hypersplenism.
• A neonatal hepatitis-like picture and massive hepatomegaly owing to steatosis have been reported. Biliary colic
secondary to cholelithiasis may occur in the 2nd decade or later.
• Liver disease occurs independent of genotype but is associated with meconium ileus and pancreatic insufficiency.

Cystic Fibrosis–Related Diabetes and Pancreatitis


• In addition to exocrine pancreatic insufficiency, evidence for hyperglycemia and glucosuria, including polyuria and
weight loss, may appear, especially in the 2nd decade of life. Ketoacidosis usually does not occur, but eye, kidney,
and other vascular complications have been noted in patients living ≥10 yr after the onset of hyperglycemia.

Genitourinary Tract
• Sexual development is often delayed but only by an average of 2 yr. More than 95% of males are azoospermic
because of failure of development of wolffian duct structures, but sexual function is generally unimpaired. The
incidence of inguinal hernia, hydrocele, and undescended testis is higher than expected. Adolescent females may
experience secondary amenorrhea, especially with exacerbations of pulmonary disease. The female fertility rate is
diminished.

Sweat Glands
• Excessive loss of salt in the sweat predisposes young children to salt depletion episodes, especially during episodes
of gastroenteritis and during warm weather.

www.onlinepgnepal.com Page 24
Online PG Nepal QAE Kartik 5

• These children present with hypochloremic alkalosis. Hyponatremia is a risk particularly in warm climates.
Frequently, parents notice salt “frosting” of the skin or a salty taste when they kiss the child.

43. All are true regarding minimal change disease except:


a. Selective proteinuria
b. hypercholesterolemia
c. IgG deposition in Mesangium
d. Respond to steroids

Ans: c
Ref: OP Ghai Pediatrics 8th Edition, Page No: 477
• Nephrotic syndrome is characterized by massive proteinuria, hypoalbuminemia and edema; hyperlipidemia is
often associated.
• Some patients show hematuria and hypertension. Heavy proteinuria (more than 1 g/m2 per day) is the underlying
abnormality, leading to hypoalbuminemia (serum albumin below 2.5 g/ dl). The resultant fall in plasma oncotic
pressure leads to interstitial edema and hypovolemia.
• Hypoalbuminemia also induces hepatic synthesis of lipoproteins resulting in hypercholesterolemia.
• Nephrotic syndrome in children can be divided into two groups based on renal histological characteristics: (i)
minimal change nephrotic syndrome (MCNS); and (ii) nephrotic syndrome with significant lesions.
• Steroid sensitive nephrotic syndrome (which is usually MCNS) has a satisfactory longterm outcome. In contrast,
the steroid resistant form (usually associated with significant glomerular lesions) has less satisfactory course and a
significant proportion progress to chronic renal failure.

STEROID SENSITIVE NEPHROTIC SYNDROME


• MCNS accounts for 80% cases of nephrotic syndrome in children.
• Renal biopsy does not show significant abnormalities on light microscopy.
• Electron microscopy shows nonspecific obliteration of epithelial foot processes.
• Immunofluorescence studies do not demonstrate deposition of immune reactants except occasional mesangial
IgM.
• On the other hand, patients with focal segmental glomerulosclerosis (FSGS) show evidence of sclerosis involving a
segment of the glomerular tuft.
• The pathogenesis of MCNS is obscure. There is evidence to suggest perturbation of cell mediated immunity, which
through yet undefined mechanisms alters the permselectivity of the glomerular filter, resulting in massive
proteinuria. A proportion of patients have a primary abnormality of the epithelial foot processes (podocytes).
Clinical Features
• The onset is insidious with edema first noticed around the eyes and subsequently on legs.
• It is soft and pits easily on pressure. Gradually edema becomes generalized, with ascites, hydrothorax and
hydrocele.
• With increasing edema, urine output may fall.
• The blood pressure is usually normal; sustained elevation suggests the possibility of significant glomerular lesions.
• The bloated appearance and relative well-being of the child is misleading and after the loss of edema, severe
muscle wasting is revealed. Infections may be present at the onset and during relapses.

44. Hypsarrhythmia is child is due to:


a. Grand mal epilepsy

www.onlinepgnepal.com Page 25
Online PG Nepal QAE Kartik 5

b. Petit mal epilepsy


c. Myoclonic epilepsy
d. Reflex epilepsy

Ans: c
Ref: OP Ghai Pediatrics 8th Edition, Page No: 559
Myoclonic Epilepsies
West syndrome (infantile spasms):
• The onset is usually between 3-8 months of life. It is characterized by a combination of salaam spells (sudden
dropping of the head and flexion of arms), developmental retardation and hypsarrhythmia on EEG.
• Common causes of infantile spasms are: (i) hypoxic ischemic encephalopathy; (ii) neurocutaneous syndromes
specially tuberous sclerosis; (iii) perinatal infections; (iv) hemorrhage;(v) injury; (vi) metabolic disorders; and (vii)
localized structural malformations; and (viii) idiopathic.
• The spasms occur in clusters usually on waking. Prognosis for normal mental development is poor. ACTH and
corticosteroids frequently help the course varies from 2-12 weeks, depending upon response. Vigabatrin is the
drug of choice, especially in tuberous sclerosis.

Lennox-Gastaut syndrome:
• Onset is usually in late infancy or childhood, is characterized by mixed seizures, including myoclonic, atypical
absence, generalized tonic-clonic or partial seizures.
• Intellectual regression is invariable. Very slow background and generalized slow and spike wave discharges (2.5
per second) are observed on EEG. This diffuse form of encephalopathy may result from factors such as head injury,
anoxia, cardiopulmonary arrest, postvaccinal encephalopathy /neurogenetic disorder and neurological infections.
• Drugs of choice are valproic acid, benzodiazepines and ACTH.
• Prognosis is often unsatisfactory. Newer antiepileptic drugs, lamotrigine, topiramate and zonisarnide are
promising.

45. All of the following are features of Juvenile CML except:


a. Thrombocytopenia
b. Lymphadenopathy
c. Philadelphia chromosome is positive
d. Fetal Hb increased

Ans: c
Ref: OP Ghai Pediatrics 8th Edition, Page No: 608
Juvenile Chronic Myeloid Leukemia
• JCML, also termed as juvenile myelomonocytic leukemia, is an uncommon malignancy accounting for less than 2%
leukemias in children.
• Patients with neurofibromatosis are at high risk for this condition. J
• CML is a disease of infancy and early childhood below the age of 5 yr, has a more acute and severe course with
frequent lymphadenopathy, anemia, hepatosplenomegaly, skin involvement (eczema, xanthoma and cafe au lait
spots), infection and thrombocytopenia.
• Peripheral smear shows leukocytosis (usually less than 1,00,0000/mm3) with the full spectrum of granulocytic
precursors and increased normoblast; monocytosis is often striking.
• Thrombocytopenia and anemia are common.

www.onlinepgnepal.com Page 26
Online PG Nepal QAE Kartik 5

• Leukocyte alkaline phosphatase score is normal or low and fetal hemoglobin levels are elevated.
• Bone marrow aspirates show an increased cellularity with predominance of granulocytic cells in all stages of
maturation; megakaryocytes are normal or decreased.
• Most patients have normal karyotype or nonspecific chromosomal abnormalities.
• Philadelphia chromosome is negative; monosomy 7 is found in 30% patients.
• JCML has a fulminant and rapidly fatal course. Management involves supportive care including packed red cell and
platelet transfusions, treatment of infections and allogeneic stem cell transplant if a matched sibling donor is
present. Even with transplant, there is 30-50% event free survival rate at 3 yr. Cis-retinoic acid has been tried with
some benefit.

46. Short fourth metatarsal is seen in:


a. Patau syndrome
b. Edward syndrome
c. Turner syndrome
d. Down syndrome

Ans: c
Ref: OP Ghai Pediatrics 8th Edition, Page No: 640-641
• Turner syndrome may be recognizable at birth lymphedema of the dorsum of hands and feet and loose skin folds
at the nape of neck. Other manifestations include short stature, short neck with webbing and low posterior
hairline.
• Anomalous ears, prominent narrow and high arched palate, small mandible and epicanthal folds may be noted.
Chest is broad shield-like with widely spaced hypoplastic Nipples.
• There is increased carrying angle at elbow. Bony anomalies include medial tibial exostosis, and short fourth
metacarpals and metatarsals. Pigmented nevi may appear when older.
• At puberty, sexual maturation fails to occur.
• The phenotype is highly variable.
• It has been recommended that the diagnosis of Turner syndrome should be considered in all girls with short
stature. Ultrasound may show streak ovaries and hypoplastic uterus. Levels of FSH and LH are increased
(hypergonadotropic hypogonadism).
• Adult stature is less than 145 cm. Associated congenital defects are common in the kidney (horseshoe kidney,
double or cleft renal pelvis), heart (coarctation of aorta) and ears (perceptive hearing defect). Congenital
lymphedema usually recedes in early infancy, leaving only puffiness over the dorsum of fingers and toes.
• Linear growth precedes at about half to three-fourths the usual rate.
• Hypothyroidism occurs in about 15-30% of adults with Turner syndrome.
• The clinical manifestations are milder in Turner syndrome with mosaicism. These patients may have normal
stature and present with secondary amenorrhea.

47. Stoss therapy is used in:


a. Scurvy
b. Keratomalacia
c. Vitamin D deficiency rickets
d. Hypophosphatemic rickets

Ans: c

www.onlinepgnepal.com Page 27
Online PG Nepal QAE Kartik 5

Ref: Nelson Textbook of Pediatrics, Elsevier, 20th Edition, Page No: 336
Treatment
Children with nutritional vitamin D deficiency should receive vitamin D and adequate nutritional intake of calcium and
phosphorus. There are 2 strategies for administration of vitamin D. With stoss therapy, 300,000-600,000
300,000 IU of vitamin D
is administered orally or intramuscularly as 22-4
4 doses over 1 day. Because the doses are observed, stoss therapy is ideal
in situations where adherence to therapy is questionable. The alternative is daily, high-dose dose vitamin D, with doses
ranging from 2,000-5,000 IU/day over 4-6 6 wk. Either strategy should be followed by daily vitamin D intake of 400 IU/day if
<1 yr old or 600 IU/day if >1 yr old. It is important to ensure that children receive adequate dietary calcium and
phosphorus; this dietary
tary intake is usually provided by milk, formula, and other dairy products.

48. Kwashiorkor is characterized by all except:


a. Alertness
b. Edema
c. Flag sign
d. Hepatomegaly

Ans: a
Ref: OP Ghai Pediatrics 8th Edition, Page No: 99
99-100
Table: difference between kwashiorkor
hiorkor and marasmus

Kwashiorkor
4 yr. The main sign is pitting edema, usually starting in the legs and feet and spreading,
It usually affects children aged 1--4
in more advanced cases, to the hands and face. Because of edema, children with kwashiorkor may m look healthy so that
their parents view them as well fed.
 General appearance: Child may have a fat sugar baby appearance.
 Edema: It ranges from mild to gross and may represent up to 55-20% 20% of the body weight.
 Muscle wasting: It is always present. The cchild
hild is often weak, hypotonic and unable to stand or walk.
 Skin changes: The skin lesions consist of increased pigmentation, desquamation and dyspigmentation.
Pigmentation may be confluent resembling flaky paint or in individual enamel spots. The distribution
distribut is typically on
buttocks, perineum and upper thigh. Petechiae may be seen over abdomen. Outer layers of skin may peel off and
ulceration may occur. The lesions may sometimes resemble burns.
 Mucous membrane lesions: Smooth tongue, cheilosis and angular stomatitis are common. Herpes simplex
stomatitis may also be seen.
 Hair: Changes include dyspigmentation, loss of characteristic curls and sparseness over temple and occipital
regions. Hairs also lose their lustre and are easily pluckable. A flag sign whic whichh is the alternate bands of
hypopigmented and normally pigmented hair pattern is seen when the growth of child occurs in spurts.

www.onlinepgnepal.com Page 28
Online PG Nepal QAE Kartik 5

 Mental changes: Includes unhappiness, apathy or irritability with sad, intermittent cry. They show no signs of
hunger and it is difficult to feed them.
 Neurological changes: These are seen during recovery.
 Gastrointestinal system: Anorexia, sometimes with vomiting, is the rule. Abdominal distension is characteristic.
Stools may be watery or semisolid, bulky with a low pH and may contain unabsorbed sugars.
 Anemia: It may also be seen, as in mild PEM, but with greater severity.
 Cardiovascular system. The findings include cold, pale extremities due to circulatory insufficiency and are
associated with prolonged circulation time, bradycardia, diminished cardiac output and hypotension.
 Renal Junction. Glomerular filtration and renal plasma flow are diminished. There is aminoaciduria and inefficient
excretion of acid load.

Obstetrics & Gynecology


49. Earliest diagnosis of pregnancy can be established safely by:
a. USG for fetal cardiac activity
b. Fetal cardiac doppler study
c. hCG levels
d. MRI pelvis

Ans: a
Ref: Williams Obstetrics 24th Edition, Page No: 196
An intrauterine gestational sac is reliably visualized with transvaginal sonography by 5 weeks, and an embryo with cardiac
activity by 6 weeks.
Cardiac motion is usually visible with transvaginal Imaging when the embryo length has reached 5 mm.

50. Drug not given in PCOD in 30 year old lady with infertility?
a. Clomiphene
b. Tamoxifen
c. OCPs
d. Metformin

Ans: b
Ref: Shaw Gynecology 16th Edition, Page No: 371
Drug not given in PCOD in a 30 year old lady with infertility is tamoxifen.
Weight loss of more than 5% of previous weight is important.
Infertility is treated with Clomiphene, 80% ovulate and 40 % conceive.
Metformin treats the root cause of PCOS, rectifies endocrine and metabolic functions and improves fertility and is drug of
choice.

51. All of the following are true about quadruple test in Down’s syndrome except:
a. Increase beta HCG
b. Decrease alpha fetoprotein
c. Decrease unconjugated estriol
d. Decrease inhibin-A

www.onlinepgnepal.com Page 29
Online PG Nepal QAE Kartik 5

Ans: d
Ref: DC Dutta Textbook of Obstetrics including Perinatology and Contraception 8th Edition, Page No: 129
Second Trimester Screening
 It is done between 15 weeks and 22 weeks.
 MSAFP: This test is done between 15 weeks and 20 weeks. MSAFP value of 2.5 multiples of the median (MOM)
when adjusted with maternal weight and ethnicity is taken as cut-off point. Elevated MSAFP detects 85% of all
neural tube defects. Cases with such high values are considered for high resolution ultrasound imaging and/or
amniocentesis. Very low MSAFP levels are associated with increased rates of miscarriage, stillbirth and neonatal
death.
 Triple Test: It is a combined biochemical test which includes MSAFP, hCG and uE3 (unconjugated estriol).
 Maternal age in relation to confirmed gestation age is also taken into account. It is used for detection of Down’s
syndrome. In an affected pregnancy, levels of MSAFP and uE3 tend to be low while that of hCG is high. It is
performed at 15–22 weeks. It gives a risk ratio and for confirmation CVS/amniocentesis has to be done. The result
is considered to be screen positive if the risk ratio is 1:250 or greater.
 Quadruple (Quad) Screening includes four biochemical analytes: (1) Maternal Serum Alpha Fetoprotein (MSAFP),
(2) Unconjugated estriol (uE3), (3) dimeric inhibin-A and (4) hCG.
 Quad screen can detect trisomy 21 in 85% of cases with a false-positive rate of 0.9%. Levels of serum analytes in
cases with trisomy 21: hCG—increased; uE3—reduced; inhibin A—elevated; MSAFP—reduced.

52. In Placenta succenturiata, if succenturiate lobe is retained it May lead to all except:
a. PPH
b. Involution
c. Polyp formation
d. Preterm delivery

Ans: d
Ref: DC Dutta Textbook of Obstetrics including Perinatology and Contraception 8th Edition, Page No: 252
PLACENTA SUCCENTURIATA
 Morphology: One (usual) or more small lobes of placenta, size of a cotyledon, may be placed at varying distances
from the main placental margin. A leash of vessels connecting the main to the small lobe traverse through the
membranes. The accessory lobe is developed from the activated villi on the chorionic laeve. In cases of absence of
communicating blood vessels, it is called placenta spuria. The incidence of placenta succenturiata is about 3%
 Diagnosis: Diagnosis is made following inspection of the placenta after its expulsion. (1) With intact lobe—the
features have already been described (2) With missing lobe: (a) there is a gap in the chorion and (b) torn ends of
blood vessels are found on the margin of the gap.
 Clinical significance: If the succenturiate lobe is retained, following birth of the placenta, it may lead to: (1)
Postpartum hemorrhage which may be primary or secondary (2) Subinvolution (3) Uterine sepsis (4) Polyp
formation.
 TREATMENT: Whenever the diagnosis of missing lobe is made, exploration of the uterus and removal of the lobe
under general anesthesia is to be done.

53. Probability of achieving live birth within a single cycle is known as:
a. Fecundity
b. Fecundability
c. Both

www.onlinepgnepal.com Page 30
Online PG Nepal QAE Kartik 5

d. None

Ans: a
Ref: DC Dutta Textbook of Gynecology including Contraception; 6th Edition, Page No: 227
 Fecundability is defined as the probability of achieving a pregnancy within one menstrual cycle. In a healthy young
couple, it is 20 percent.
 Fecundity is the probability of achieving a live birth within a single cycle.

54. External os protrudes outside the vaginal introitus in which degree of uterine prolapse?
a. 1st
b. 2nd
c. 3rd
d. 4th

Ans: b
Ref: DC Dutta Textbook of Gynecology including Contraception; 6th Edition, Page No: 205 - 206
Three degrees of uterine prolapse are described:
 First degree — The uterus descends down from its normal anatomical position (external os at the level of ischial
spines) but the external os still remains inside the vagina.
 Second degree — The external os protrudes outside the vaginal introitus but the uterine body still remains
inside the vagina
 Third degree (Syn: Procidentia, Complete prolapse)— The uterine cervix and body descends to lie outside the
introitus. Procidentia involves prolapse of the uterus with eversion of the entire vagina. Complex prolapse is one
when prolapse is associated with some other specific defects.

55. What is irregular, acyclic bleeding from uterus is known as?


a. Menorrhagia
b. Metrorrhagia
c. Polymenorrhea
d. Hypomenorrhea

Ans: b
Ref: DC Dutta Textbook of Gynecology including Contraception; 6th Edition, Page No: 185 – 186
 Menorrhagia is defined as cyclic bleeding at normal intervals; the bleeding is either excessive in amount (> 80 mL)
or duration (>7 days) or both.
 Polymenorrhea is defined as cyclic bleeding where the cycle is reduced to an arbitrary limit of less than 21 days
and remains constant at that frequency. If the frequent cycle is associated with excessive and or prolonged
bleeding, it is called epimenorrhagia.
 Metrorrhagia is defined as irregular, acyclic bleeding from the uterus. Amount of bleeding is variable.
 Menstrual bleeding occurring more than 35 days apart and which remains constant at that frequency is called
oligomenorrhea.
 When the menstrual bleeding is unduly scanty and lasts for less than 2 days, it is called hypomenorrhea.

56. Commonest fibroid is:


a. Intramural

www.onlinepgnepal.com Page 31
Online PG Nepal QAE Kartik 5

b. Subserous
c. Submucosal
d. None

Ans: a
Ref: DC Dutta Textbook of Gynecology including Contraception; 6th Edition, Page No: 272

57. Biparietal diameter of fetal skull is:


a. 7.5 cm
b. 8 cm
c. 8.5 cm
d. 9.5 cm

Ans: d
Ref: DC Dutta Textbook of Obstetrics including Perinatology and Contraception 8th Edition, Page No: 96
 Biparietal diameter—9.5 9.5 cm (3 ¾"): It extends between two parietal eminences. Whatever maybe m the position
of the head, this diameter nearly always engages.
 Super-subparietal—8.5 8.5 cm (3 ½"): It extends from a point placed below one parietal eminence to a pointplaced
above the other parietal eminence of the opposite side.
 Bitemporal diameter—8 cm m (3 ¼"): It is the distance between the anteroinferior ends of the coronal suture.
 Bimastoid diameter — 7.5 cm (3"): It is the distance between the tips of the mastoid processes. The diameter is
incompressible and it is impossible to reduce the length of the bimastoid diameter by obstetrical operation

58. Gonococcal vaginitis occurs in:


a. Adults
b. Middle age
c. Infants
d. Adolescents

Ans: c

www.onlinepgnepal.com Page 32
Online PG Nepal QAE Kartik 5

Ref: DC Dutta Textbook of Gynecology including Contraception; 6th Edition, Page No: 147
 As squamous epithelium is resistant to gonococcal invasion, vaginitis in adult is not possible, but vulvovaginitis is
possible in childhood. In about 15 percent of untreated cervicitis, gonococcal infection may ascend up to produce
acute pelvic inflammatory disease (PID). Rarely, it may pro
produce
duce septicemia with distant involvement to cause
tenosynovitis and septic arthritis. Upper genital organs are involved as the infection spreads along the
spermatozoa.
 Gonococci attach to the spermatozoa and are being carried up.
 Endometritis and salpingitistis are common.
 It should be remembered that N. gonorrhoeae is often present with other sexually transmitted diseases and
women with gonorrhea are considered to be at risk for incubating syphilis. One
One-third
third of such cases are associated
with chlamydial infection.

59. The major contribution to the human seminal fluid is from:


a. Testes
b. Seminal vesicles
c. Prostate
d. Bulbourethral and urethral glands

Ans: b
Ref: DC Dutta Textbook of Gynecology including Contraception; 6th Edition, Page No: 232
The seminal vesicle contributes
ntributes 60% and prostate about 30% of the seminal fluid.

60. Halban’s disease is due to:


a. Persistent corpus luteum
b. Deficient corpus luteum
c. Persistent trophoblast
d. Deficient trophoblast

www.onlinepgnepal.com Page 33
Online PG Nepal QAE Kartik 5

Ans: a
Ref: Shaw Gynecology 16th Edition, Page No: 347
Irregular Shedding (Halban’s Disease)
• It is rare and self-limited. Irregular shedding is due to persistent corpus luteum.
• The menstruation comes on time, is prolonged but not heavy.
• Progestogen can suppress the bleeding, but needs to be taken on tapering dose for 20 days to complete the cycle

Orthopedics
61. Where does the head of humerus commonly lie in anterior shoulder dislocation?
a. Preglenoid
b. Subclavicular
c. Subcoracoid
d. Subglenoid

Ans: C, Subcoracoid
Ref: Maheshwari Orthopedics, 5th edition,page 90
Dislocations of the shoulder may be of the following types:
 a) Anterior dislocation: In this injury, the head of the humerus comes out of the glenoid cavity and lies anteriorly.
It may be further classified into three subtypes depending on the position of the dislocated head.
o Preglenoid: The head lies in front of the glenoid.
o Subcoracoid: The head lies below the coracoid process. Most common type of dislocation.
o Subclavicular: The head lies below the clavicle.
 b) Posterior dislocation: In this injury, the head of the humerus comes to lie posteriorly, behind the glenoid.
 c) Luxatio erecta (inferior dislocation): This is a rare type, where the head comes to lie in the subglenoid position.

62. What is the commonest site of avascular necrosis of scaphoid after fracture through the waist of the scaphoid?
a. Distal pole of scaphoid
b. Lateral pole of scaphoid
c. Medial pole of scaphoid
d. Proximal pole of scaphoid

Ans: D, Proximal pole of scaphoid


Ref: Maheshwari Orthopedics, 5th edition,page 50
Common sites of avascular necrosis:
o Head of the femur: Fracture neck of the femur. Posterior dislocation of the hip
o Proximal pole of scaphoid: Fracture through the waist of the scaphoid
o Body of the talus: Fracture through neck of the talus

63. What is the position in which arthrodesis of wrist joint is done?


a. Dorsiflexion 20
b. Extension 40
c. Volar flexion 30
d. Neutral position

www.onlinepgnepal.com Page 34
Online PG Nepal QAE Kartik 5

Ans: A, Dorsiflexion 20
Ref: Maheshwari Orthopedics, 5th edition,page 84

64. What is commonly seen as monoarthritis?


a. Primary osteoarthritis
b. Rheumatoid arthritis
c. Seronegative spondarthritis
d. Tubercular arthritis

Ans: D, Tubercular arthritis


Ref: Maheshwari Orthopedics, 5th edition,page 286
Types of arthritis
Monoarthritis
 Pyogenic arthritis
 Tubercular arthritis
 Haemophilic arthritis
 Secondary osteoarthritis
 Gout - sometimes
Polyarthritis
 Rheumatoid arthritis
 Rheumatic fever
 Juvenile chronic polyarthritis
 Primary osteoarthritis
 Seronegative spondarthritis

65. What is the common site of occurrence of osteoclastoma?


a. Diaphysis
b. Epiphysis
c. Metaphysis
d. Anywhere

Ans: B, Epiphysis

www.onlinepgnepal.com Page 35
Online PG Nepal QAE Kartik 5

Ref: Maheshwari Orthopedics, 5th edition,page 244

66. What is defective in osteogenesis imperfecta?


a. Collagen synthesis
b. Calcification
c. Osteoclastic remodeling
d. Phosphate deposition

Ans: A, Collagen synthesis


Ref: Maheshwari Orthopedics, 5th edition, page 316
 Osterogenesis imperfecta is a condition characterized by tendency for frequent fractures, usually with minimal
trauma because of weak and brittle bones.
 It results from defective collagen synthesis, and thus affects bone as well as other collagen containing
cont structures
such as the skin, sclera, teeth, ligaments etc.

67. Space of Poirier is associated with anterior dislocation of which of the following carpal bone?
a. Hamate
b. Lunate
c. Scaphoid
d. Trapezoid

Ans: B, Lunate
paedics & Trauma, 10th edition, page 428
Ref: Apley & Solomon’s System of Orthopaedics
Space ofPoirier is the gap between the lunate and midcarpal joint through which the lunate can dislocate anteriorly.

68. Which of the following is a joint X-ray


ray change seen in doubtful osteoarthritis?
a. Definite osteophyte
b. Diminution of joint space

www.onlinepgnepal.com Page 36
Online PG Nepal QAE Kartik 5

c. Minimal osteophyte
d. Sclerosis of bone

Ans: C, Minimal osteophyte


Ref: Apley & Solomon’s System of Orthopaedics & Trauma, 10th edition, page 96
As with the pathological changes, there are a number of different scoring syste
systems
ms available to assess the severity of
radiographic changes. The one used most commonly is the Kellgren and Lawrence scoring system, which divides OA X-ray
X
changes into five categories:
 0: Normal: No features of OA
 1: Doubtful: Minimal osteophyte, doubtful significance
 2: Minor: Definite osteophyte, no loss of joint space
 3: Moderate: Some diminution of joint space
 4: Severe: Advanced joint space loss and sclerosis of bone

69. Which nerve is injured in crutch palsy?


a. Axillary nerve
b. Median nerve
c. Radial nerve
d. Ulnar nerve

Ans: C, Radial nerve


Ref: Maheshwari Orthopedics, 5th edition, page 74
Crutch palsy is injury to the radial nerve.

70. What is responsible for lateral collateral ligament injury?


a. Backward force on tibia
b. Twisting force
c. Valgus force
d. Varus force

Ans: D, Varus force


Ref: Maheshwari Orthopedics, 5th edition, page 149

ENT
71. What is not a characteristic feature of sensorineural hearing loss?
a. Involving high frequencies
b. No difficulty in hearing in the presence of noise

www.onlinepgnepal.com Page 37
Online PG Nepal QAE Kartik 5

c. Positive Rinne test


d. Weber lateralized to better ear

Ans: B, No difficulty in hearing in the presence of noise


Ref: Dhingra ENT, 7th edition, page 34
Sensorineural hearing loss (SNHL) results from lesions of the cochlea, VIIIth nerve or central auditory pathways. It may be
present at birth (congenital) or start later in life (acquired).
The characteristics of sensorineural hearing loss are:
o A positive Rinne test, i.e. AC > BC.
o Weber lateralized to better ear.
o Bone conduction reduced on Schwabach and absolute bone conduction tests.
o More often involving high frequencies.
o No gap between air and bone conduction curve on audiometry
o Loss may exceed 60 dB.
o Speech discrimination is poor.
o There is difficulty in hearing in the presence of noise.

72. What is the nerve supply of tensor veli palatini muscle?


a. V
b. IX
c. X
d. XI

Ans: A, V
Ref: Dhingra ENT, 7th edition, page 61
 Tympanic branch of cranial nerve (CN) IX supplies sensory as well as parasympathetic secretomotor fibres to the
tubal mucosa.
 Tensor veli palatini muscle is supplied by mandibular branch of trigeminal (V3) nerve.
 Levator veli palatini and salpingopharyngeus muscles receive motor nerve supply through pharyngeal plexus
(cranial part of CN XI through vagus).

73. What does cholesteatoma contain within its center?


a. Fibrous stroma
b. Keratin debris
c. Squamous epithelium
d. Wax

Ans: B, Keratin debris


Ref: Dhingra ENT, 7th edition, page 73
Essentially, cholesteatoma consists of two parts:
 The matrix, which is made up of keratinizing squamous epithelium resting on a thin stroma of fibrous tissues and
 a central white mass, consisting of keratin debris produced by the matrix. For this reason, it has also been named
epidermosis or keratoma.

74. What is not true about Ménière’s disease?

www.onlinepgnepal.com Page 38
Online PG Nepal QAE Kartik 5

a. Constant hearing loss


b. Episodic vertigo
c. Fullness of ear
d. Tinnitus

Ans: A, Constant hearing loss


Ref: Dhingra ENT, 7th edition, page 111 – 112
Cardinal symptoms of Ménière’s disease are
o episodic vertigo,
o fluctuating hearing loss,
o tinnitus, and
o sense of fullness or pressure in the involved ear.

75. What is occipitofrontal view of paranasal sinuses also known?


a. Basal view
b. Caldwell view
c. Lateral view
d. Waters’ view

Ans: B, Caldwell view


Ref: Dhingra ENT, 7th edition, page 493 – 494
Paranasal sinuses are visualized radiologically through:
Waters’ view (occipitomental view or nose-chin position). It is taken in such a way that nose and chin of the patient touch
the film while X-ray beam is projected from behind. Waters’ view with open mouth is preferred as it also shows sphenoid
sinus. In this view, petrous bones are projected below the maxillary antra. Structures seen are:
o (a) Maxillary sinuses (seen best).
o (b) Frontal sinuses.
o (c) Sphenoid sinus (if the film is taken with open mouth).
o (d) Zygoma.
o (e) Zygomatic arch.
o (f) Nasal bone.
o (g) Frontal process of maxilla.
o (h) Superior orbital fissure.
o (i) Intratemporal fossa

Caldwell view (Occipitofrontal view or nose-forehead position). The view is taken with nose and forehead touching the
film and X-ray beam is projected 15–20° caudally. Structures seen are:
o (a) Frontal sinuses (seen best).
o (b) Ethmoid sinuses.
o (c) Maxillary sinuses.
o (d) Frontal process of zygoma and zygomatic process of frontal bone.
o (e) Superior margin of orbit and lamina papyracea.
o (f) Superior orbital fissure.
o (g) Foramen rotundum (inferolateral to superior orbital fissure).

www.onlinepgnepal.com Page 39
Online PG Nepal QAE Kartik 5

Lateral view. Lateral side of the skull lies against the film and X-ray beam is projected perpendicular from the other side.
Structures seen are:
o (a) Anterior and posterior extent of sphenoid, frontal and maxillary sinuses.
o (b) Sella turcica.
o (c) Ethmoid sinuses.
o (d) Alveolar process.
o (e) Condyle and neck of mandible.

Submentovertical (basal) view. This view is taken as described earlier. Structures seen are:
o (a) Sphenoid, posterior ethmoid and maxillary sinuses (seen best in that order).
o (b) Zygoma.
o (c) Zygomatic arch.
o (d) Mandible along with coronoid and condyloid processes.

Right and left oblique views. They are taken to see the posterior ethmoid sinuses and the optic foramen of the
corresponding side.

76. Which artery doesn’t supply the Little’s area?


a. Anterior ethmoidal artery
b. Posterior ethmoidal artery
c. Septal branch of sphenopalatine artery
d. Septal branch of superior labial artery

Ans: B, Posterior ethmoidal artery


Ref: Dhingra ENT, 7th edition, page 197
 Little’s area is situated in the anterior inferior part of nasal septum, just above the vestibule. Four arteries –
anterior ethmoidal, septal branch of superior labial, septal branch of sphenopalatine and the greater palatine,
anastamose here to form a vascular plexus known as “Kiesselbach’s plexus”.
 Woodruff’s plexus is a plexus of veins situated inferior to posterior end of inferior turbinate. It is the site of
posterior epistaxis in adults.

www.onlinepgnepal.com Page 40
Online PG Nepal QAE Kartik 5

77. What is a common cause of bilateral nasal obstruction?


a. Collapsing nasal alae
b. Furuncle
c. Nasoalveolar cyst
d. Squamous cell carcinoma

Ans: A, Collapsing nasal alae


Ref: Dhingra ENT, 7th edition, page 194
Common causes of bilateral nasal obstruction
Vestibule
o Bilateral vestibulitis
o Collapsing nasal alae
o Stenosis of nares
o Congenital atresia of nares
Nasal cavity
o Acute rhinitis (viral and bacterial)
o Chronic rhinitis and sinusitis
o Rhinitis medicamentosa
o Allergic rhinitis
o Hypertrophic turbinates
o DNS
o Nasal polypi
o Atrophic rhinitis
o Rhinitis sicca
o Septal haematoma
o Septal abscess
o Bilateral choanal atresia
Nasopharynx

www.onlinepgnepal.com Page 41
Online PG Nepal QAE Kartik 5

o Adenoid hyperplasia
o Large choanal polyp
o Thornwaldt’s cyst
o Adhesions between soft palate and posterior pharyngeal wall
o Large benign and malignant tumours

78. What is not a malignant nasopharyngeal tumor?


a. Chordoma
b. Craniopharyngioma
c. Haemangiopericytoma
d. Plasmacytoma

Ans: B, Craniopharngioma
Ref: Dhingra ENT, 7th edition, page 279
Benign and malignant tumours of the nasopharynx:
Benign
 Angiofibroma
 Choanal polyp
 Squamous papilloma
 Thornwaldt’s cyst
 Pleomorphic adenoma
 Craniopharyngioma
 Paraganglioma
 Hamartoma
 Congenital tumours
 Hairy polyp
 Teratoma
 Epignathi
Malignant
 Carcinoma nasopharynx
 Lymphoma
 Rhabdomyosarcoma
 Chordoma
 Plasmacytoma
 Haemangiopericytoma
 Malignant salivary glandtumours
 Melanoma

79. What is an extrinsic muscle of larynx?


a. Cricothyroid
b. Sternothyroid
c. Thyroepiglottic
d. Vocalis

Ans: B, Sternothyroid

www.onlinepgnepal.com Page 42
Online PG Nepal QAE Kartik 5

Ref: Dhingra ENT, 7th edition, page 320


Intrinsic muscles of larynx acting on vocal cords are:
 Abductors: Posterior cricoarytenoid
 Adductors: Lateral cricoarythenoid, interarytenoid and thyroarytenoid
 Tensors: Cricothyroid and vocalis
Intrinsic muscles of larynx acting on laryngeal inlet are:
 Openers: Thyroepiglottic
 Closers: Interarytenoid and aryepiglottic
Extrinsic muscles are:
 Elevators: Primary elevators are stylopharyngeus, salpingopharyngeus, palatopharyngeus and thyrohyoid.
Secondary elevators are mylohyoid (main), digastric, stylohyoid and geniohyoid
 Depressors: Sternohyoid, sternothyroid and omohyoid

80. What is involved in Ludwig’s angina?


a. Carotid space
b. Parapharyngeal space
c. Retropharyngeal space
d. Submandibular space

Ans: D, Submandibular space


Ref: Dhingra ENT, 7th edition, page 297
Submandibular space lies between mucous membrane of the floor of mouth and tongue on one side and superficial layer
of deep cervical fascia extending between the hyoid bone and mandible on the other. It is divided into two compartments
by the mylohyoid muscle:
 Sublingual compartment (above the mylohyoid).
 Submaxillary and submental compartment (below the mylohyoid).
 The two compartments are continuous around the posterior border of mylohyoid muscle.
Ludwig’s angina is infection of submandibular space.

Ophthalmology
81. Where is coloboma iris located?
a. Inferonasal
b. Inferotemporal
c. Superonasal
d. Superotemporal

Ans: A, Inferonasal
Ref: Khurana Ophthalmology, 6th edition, page 149
 Congenital coloboma (absence of tissue) of iris, ciliary body and choroid may be seen in association or
independently. Coloboma may be typical or atypical.
 Typical coloboma is seen in the inferonasal quadrant and occurs due to defective closure of the embryonic fissure.
 Atypical coloboma is occasionally found in other positions.

82. Circumcorneal zone of conjunctiva is supplied by branches of which of the following nerves?

www.onlinepgnepal.com Page 43
Online PG Nepal QAE Kartik 5

a. Frontal
b. Infratrochlear
c. Long ciliary nerve
d. Supratrochlear

Ans: C, Long ciliary nerve


Ref: Khurana Ophthalmology, 6th edition, page 61
Nerve supply of conjunctiva
 A circumcorneal zone of conjunctiva is suppliedby the branches from long ciliary nerves whichsupply the cornea.
 Rest of the conjunctiva is supplied by the branchesfrom lacrimal, infratrochlear, supratrochlear,supraorbital and
frontal nerves.

83. What is false about macula lutea?


a. About 5.5 mm in diameter
b. Also known as the yellow spot
c. It is the central depressed part of fovea centralis
d. Situated at the posterior pole temporal to the optic disc

Ans: C, It is the central depressed part of fovea centralis


Ref: Khurana Ophthalmology, 6th edition, page 263
Macula lutea is also called the yellow spot. It is comparatively deeper red than the surrounding fundus and is situated at
the posterior pole temporal to the optic disc. It is about 5.5 mm in diameter. Fovea centralis is the central depressed part
of the macula. It is about 1.5 mm in diameter and is the most sensitive part of the retina.

84. What is the most common cause of bilateral proptosis in an adult?


a. Craniofacial dysostosis
b. Neuroblastoma
c. Retinoblastoma
d. Thyroid ophthalmopathy

Ans: D, Thyroid ophthalmopathy


Ref: Khurana Ophthalmology, 6th edition, page 404
Most common cause of bilateral proptosis in children is neuroblastoma and leukemia (chloroma) and in adults is thyroid
ophthalmopathy.

85. What is false about orbital apex syndrome?


a. CSF rhinorrhea
b. Ipsilateral ophthalmoplegia
c. Ptosis
d. Retro-orbital pain

Ans: A, CSF rhinorrhea


Ref: Khurana Ophthalmology, 6th edition, page 413
Orbital apex syndrome refers to the symptom complex produced by involvement of structures present at the apex of the
orbit, i.e., structures passing through the superior orbital fissure and the optic canal.

www.onlinepgnepal.com Page 44
Online PG Nepal QAE Kartik 5

So features of orbital apex syndrome include:


Features of superior orbital fissure:
o Pain, typically labeled as retro-orbital ache
o Sensory loss/disturbances in the distribution of ophthalmic division of 5th cranial nerve
o Ipsilateral ophthalmoplegia due to 3rd, 4th and 6th cranial nerve involvement
o Ptosis due to 3rd cranial nerve paralysis
Features of optic nerve involvement: early visual loss and afferent pupillary defect

86. Which of the following is not a feature of pathological myopia?


a. Large and pale optic disc
b. Prominent eyeballs
c. Shallow anterior chamber
d. Sluggishly reacting pupils

Ans: C, Shallow anterior chamber


Ref: Khurana Ophthalmology, 6th edition, page 40 – 41
 Pathological/degenerative/progressive myopia, as the name indicates, is a rapidly progressive error which starts in
childhood at 5–10 years of age and results in high myopia (>–6D) during early adult life which is usually associated
with degenerative changes in the eye. It is less common (about 2% of population).
 The symptoms are defective vision, muscae volitantes and night blindness.
 Signs are prominent eyeballs, large cornea, deep anterior chamber. Pupils are slightly large and react sluggishly to
light.
o Fundus examination reveals following characteristic signs: Optic disc appears large and pale and at its
temporal edge a characteristic myopic crescent is present. Sometimes peripapillary crescent encircling the
disc may be present, where the choroid and retina is distracted away from the disc margin. A super-
traction crescent (where the retina is pulled over the disc margin) may be present on the nasal side.
o Degenerative changes in retina and choroid are common in progressive myopia. These are characterized
by: Chorioretinal atrophic patches at the macula with a little heaping up of pigment around them; Foster-
Fuchs’ spot (dark red circular patch due to sub-retinal neovascularization and choroidal haemorrhage)
may be present at the macula; Cystoid degeneration may be seen at the periphery. Lattice degeneration
and or snail track lesions with or without retinal holes/tears may be present; which later may be
complicated by retinal detachment; total retinal atrophy, particularly in the central area may occur in an
advanced case.
o Posterior staphyloma due to ectasia of sclera at posterior pole may be apparent as an excavation with the
vessels bending backward over its margins.
o Degenerative changes in vitreous include: liquefaction, vitreous opacities, and posterior vitreous
detachment (PVD) appearing as Weiss’ reflex.
o Visual fields may show contraction and in some cases ring scotoma may be seen.
o ERG may reveal subnormal electroretinogram due to chorioretinal atrophy.

87. What is false about aphakia?


a. Hypermetropic small disc
b. Iridonesis
c. Shallow anterior chamber
d. Two images seen in Purkinje test

www.onlinepgnepal.com Page 45
Online PG Nepal QAE Kartik 5

Ans: C, Shallow anterior chamber


Ref: Khurana Ophthalmology, 6th edition, page 37
Aphakia literally means ‘absence of crystalline lens’ from the eye. However, from the optical point of view, it may be
considered a condition in which the lens is absent from the pupillary area.
Signs of aphakia:
o Limbal scar may be seen in surgical aphakia.
o Anterior chamber is deeper than normal.
o Iridodonesis i.e., tremulousness of iris can be demonstrated.
o Pupil is jet black in colour.
o Purkinje’s image test shows only two images (normally four images are seen).
o Fundus examination shows hypermetropic small disc.
o Retinoscopy and autorefractometry reveals high hypermetropia

Anesthesiology
88. Triad of general anesthesia includes all except?
a. Amnesia
b. Analgesia
c. Muscle relaxation
d. Narcosis

Ans: A, Amnesia
Ref: Ajay Yadav Anesthesia, 6th edition, page 87
The basic components (Triad) of general anesthesia are:
o Narcosis
o Analgesia
o Muscle relaxation
The anesthesia in present day should be balanced anesthesia, a concept laid by John Lundy in 1926. The components of
balance anesthesia are:
o Narcosis
o Analgesia
o Muscle relaxation
o Amnesia
o Abolition of reflexes
o Maintenance of normal hemodynamics and physiologic homoeostasis

89. What does not increase the anatomical dead space?


a. Atropine
b. Bronchodialtors
c. Jaw protrusion
d. Neck flexion

Ans: D, Neck flexion


Ref: Ajay Yadav, Short Textbook of Anesthesia, 6th edition, page 7

www.onlinepgnepal.com Page 46
Online PG Nepal QAE Kartik 5

Anatomical dead space is constituted by air which is not participating in diffusion. Therefore, it is constituted by air
present in nose, trachea and bronchial tree (up to terminal bronchioles). Normally, it is 30% of tidal volume or 2 mL/kg or
150 mL.
Anatomical dead space is increased in:
 Old age
 Neck extension
 Jaw protrusion
 Bronchodilators
 Increasing lung volume
 Atropine (causes bronchodilatation)
 Anesthesia mask, circuits
 Intermittent positive pressure ventilation (IPPV) and positive end-expiratory pressure (PEEP).
Anatomical dead space is decreased by:
 Intubation (nasal cavity is bypassed and diameter of tube is less than airway diameter)
 Tracheostomy (upper airways and nasal cavity bypassed)
 Hyperventilation (decreasing lung volume)
 Neck flexion
 Bronchoconstrictor

90. Which anesthetic agent produces the lowest cerebral vasodilatation?


a. Desflurane
b. Halothane
c. Isoflurane
d. Sevoflurane

Ans: D, Sevoflurane
Ref: Ajay Yadav Anesthesia, 6th edition, page 224
 All inhalational agents (including nitrous oxide) increases the cerebral blood flow and hence ICT.
 The order of vasodilation potency is- halothane >> Desflurane = lsoflurane > Sevoflurane.

91. How does dexmedetomidine exert its effect?


a. α1 adrenergic agonist
b. α1 adrenergic antagonist
c. α2 adrenergic agonist
d. α2 adrenergic antagonist

Ans: C, α2 adrenergic agonist


Ref: Miller’s Anesthesia, 8th edition, page 822
 Dexmedetomidine is the most recently released IV anesthetic. It is a highly selective α2-adrenergic agonist that
produces sedation, sympatholysis, hypnosis, and analgesia. Its primary action is as an agonist on α2 receptors in
the locus coeruleus. It has minimal effect on respiration. Heart rate and cardiac output show a concentration
dependent decrease.

www.onlinepgnepal.com Page 47
Online PG Nepal QAE Kartik 5

 Dexmedetomidine is currently approved only for brief (<24 hours) postoperative sedation. Its use as an adjunct or
sole hypnotic agent is rapidly emerging, most frequently in an ICU setting. It may be advantageous for its ability to
prevent delirium.
 Dexmedetomidine is used as a sedative during invasive or radiologic procedures and as an adjunct in central or
peripheral neural blockade.

92. Noninvasive positive pressure ventilation indicated in all except:


a. Conscious patient
b. Cooperative patient
c. Patient with facial trauma
d. Patient without risk of aspiration

Ans: C, Patient with facial trauma


Ref: Ajay Yadav Anesthesia, 6th edition, page 282
Since intubation and tracheostomy carries their multitude of complications; therefore, whenever possible, a trial of
noninvasive positive pressure ventilation (NIPPY) should be given. Ideal candidates for trial of NIPPY are:
 The patients who are in respiratory failure but there is no urgent need of intubation
 Conscious and cooperative patients
 Patients in whom there is no risk of aspiration
 Face mask could be tightly fitted
NIPPY is contraindicated in:
 Cardiac or respiratory arrest
 Severe hypoxemia
 High risk of aspiration
 Facial trauma
 Inability to protect airways
 Upper GI bleed

93. Which inhalational anesthetic agent is a pulmonary vasoconstrictor?


a. Halothane
b. Isoflurane
c. Nitrous oxide
d. Xenon

Ans: C, Nitrous oxide


Ref: Ajay Yadav Anesthesia, 6th edition, page 92
All inhalational agents are pulmonary vasodilators (except nitrous oxide which is pulmonary vasoconstrictor).

94. What is the most accurate method to confirm an endotracheal intubation?


a. Auscultation
b. Capnography
c. Fogging of tube
d. X-ray

Ans: B, Capnography

www.onlinepgnepal.com Page 48
Online PG Nepal QAE Kartik 5

Ref: Morgan Anesthesia, 4th edition, page 590


 Capnography is the continuous measurement of end tidal carbon dioxide. It is the surest confirmatory sign of
correct intubation.
 It can also be used to diagnose intraoperative displacement of endotracheal tube, malignant hyperthermia, for
detection of obstruction/disconnection of ET tube, pulmonary embolism, as an aid during Cardio Pulmonary
Resuscitation.

Dermatology
95. What type of hair loss presents with recession of the frontal hairline in a triangular pattern?
a. Alopecia areata
b. Androgenetic alopecia
c. Lichen planopilaris
d. Telogen effluvium

Ans: B, Androgenetic alopecia


Ref: Fitzpatrick’s Dermatology, 9th edition, page 1495 – 1496
 Androgenetic alopecia (AGA) is the most common type of hair loss, a nonscarring progressive miniaturization of
the hair follicle with shortening of the anagen phase in genetically predisposed men and women, usually in a
specific pattern distribution.
 Recession of the frontal hairline, mainly in a triangular pattern is the characteristic finding, later followed by a
vertex thinning with progression until the top of the scalp is completely bald. Occipital area and sides of the scalp
are spared even in longstanding male pattern hair loss.

96. Which of the following skin disease produces nail changes most commonly?
a. Atopic eczema
b. Lichen planus
c. Pompholyx
d. Psoriasis

Ans: D, Psoriasis
Ref: Fitzpatrick Dermatology, 9th edition, page 1578
 Psoriasis is the skin disease with the most frequent nail involvement. At the time of consultation, approximately
50% of the patients present with nail changes. Over their lifetime, up to 90% of all psoriatics will develop nail
alterations.
 The prevalence is even higher in psoriatic arthritis.

97. What is incorrect about pemphigus vulgaris?


a. Desmoglein 1 and 3 are the involved antigens
b. DIF shows IgG in intercellular fishnet membrane
c. Nikolsky sign positive
d. Tense blisters present

Ans: D, Tense blisters present


Ref: Fitzpatrick’s Dermatology, 9th edition, page 951

www.onlinepgnepal.com Page 49
Online PG Nepal QAE Kartik 5

98. A female who was receiving lamotrigine, phenytoin and carbamazepine for new onset seizures presented with
wide spread purpuric macules, erosions over genitalia and hemorrhagic crusts over lips along with detachment of 35%
of skin of her body, a week after starting medications. What is the diagnosis?
a. Drug reaction with eosinophilia and systemic symptoms
b. SJS-TEN overlap
c. Stevens Johnson syndrome
d. Toxic epidermal necrolysis

Ans: D, Toxic epidermal necrolysis


Ref: Rook’s Dermatology, 9th edition, page 119.17
 SJS is defined as: epidermal detachment less than 10% BSA, plus widespread purpuric macules or flat atypical
targets.
 Overlap SJS-TEN: detachment of 10–30% 30% BSA, plus widespread purpuric macules or flat atypical targets.
 TEN with spots: detachment greater than 30% BSA, plus wide
wide-spread
spread purpuric macules or flat atypical targets.
 TEN without spots: detachment greater th than
an 30% BSA, with loss of large epidermal sheets without purpuric
macules or target lesions.

www.onlinepgnepal.com Page 50
Online PG Nepal QAE Kartik 5

99. Which of the following when present is enough to make a diagnosis of systemic sclerosis?
a. Raynaud phenomenon
b. Scleroderma-related
-related antibodies present in sufficiently high titers
c. Skin thickening of the fingers along with fingertip ulcers
d. Skin thickening of the fingers of both handsextending proximal to the MCP joints

Ans: D, Skin thickening of the fingers of both hands extending proximal to the MCP joints
Ref: Rook’s Dermatology, 9th edition, page 56.2

100. Which layer of epidermis is thickened in lichen planus to form Wickham striae?
a. Basal
b. Corneum
c. Granular
d. Spinosum

Ans: C, Granular
Ref: Bolognia Dermatology, 4th edition, page 192
 The characteristic primary lesion of Lichen Planus is a small, polygonal
polygonal-shaped,
shaped, violaceous, flat-topped
flat papule;
occasionally papules are umbilicated.
 The surface is slightly shiny or transparent, and a network of fine white lines called “Wickham striae” or small
gray–white puncta are also seen. The latter correspond histologically to focal thickening of the granular layer.
Wickham striae are readily apparent by dermoscopy.

101. What is the approximate percentage of UV blockage provided by well applied sunscreen of SPF 30?

www.onlinepgnepal.com Page 51
Online PG Nepal QAE Kartik 5

a. 15%
b. 30%
c. 70%
d. 97%

Ans: D, 97%
Ref: Bolognia Dermatology, 4th edition, page 2311

Psychiatry
102. Not true about characteristic feature of Tourette syndrome is:
a. Multiple motor tics
b. Multiple vocal tics
c. Duration > 6 months
d. More common in males

Ans: c
Ref: Niraj Ahuja Psychiatry 7th Edition, Page No: 168
Tourette’s disorder is characterized by:
 Multiple motor tics
 Multiple vocal tics
 Duration > 1 year
 Onset usually before 11 years of age and almost always before 21 years of age.
 The disorder is about 3 times more commo
common n in males and has a prevalence rate of about 0.5 per 1000 population.

103. Asperger's syndrome is:


a. Metabolic disorder
b. Developmental disorder

www.onlinepgnepal.com Page 52
Online PG Nepal QAE Kartik 5

c. Thought disorder
d. Mood disorder

Ans: b
Ref: Niraj Ahuja Psychiatry 7th Edition, Page No: 165
Autism spectrum disorders (ASD) encompasses
• Autistic disorder
• Childhood disintegrative disorder
• Pervasive developmental disorder-not otherwise specified
• Asperger disorder

DSM V criteria
Social communication and social interaction deficit in many contexts such as
• lack of social-emotional reciprocity
• lack of nonverbal communicative behaviors
• impairment in developing, maintaining, and understanding relationships
Restricted and repetitive patterns of behavior, interests, or activities such as
• motor movements that are stereotyped or repetitive (e.g., flipping objects)
• inflexibility to change
• restricted and fixated interests; these are typically with abnormal intensity or focus
• hyper- or hyporeactivity or unusual interest in a sensory stimulus (e.g., fascination with lights)
Symptoms must be present in the patient's early developmental period in the absence of an organic etiology (e.g., hearing
dysfunction)
Symptoms cannot be better explained by other conditions (e.g., intellectual developmental disorder)

104. Which drug is the most useful in treating an episode of antipsychotic induced acute dystonia?
a. Lorazepam
b. Haloperidol
c. Promethazine
d. Phenobarbitone

Ans: c
Ref: KD Tripathi Essentials of Medical Pharmacology; 7th Edition, Page No: 444
Acute muscular dystonias
• Bizarre muscle spasms, mostly involving linguo-facial muscles —grimacing, tongue thrusting, torticollis, locked jaw;
occurs within a few hours of a single dose or at the most in the first week of therapy. It is more common in
children below 10 years and in girls, particularly after parenteral administration; overall incidence is 2%. It lasts for
one to few hours and then resolves spontaneously.
• One of the central anticholinergics, promethazine or hydroxyzine injected i.m. clears the reaction within 10–15
min.

105. Which of these is a mood stabilizer with an anti-suicidal effect?


a. Lamotrigine
b. Carbamazepine
c. Lithium

www.onlinepgnepal.com Page 53
Online PG Nepal QAE Kartik 5

d. Valproate

Ans: c
Ref: Kaplan and Sadock Psychiatry 11th Edition, Page No: 236
 Lithium is the drug of choice for Bipolar disorders as it is a mood stabilizer as well as it has an anti-suicidal effect.
Hence it is also used as a response augmenting agent with antidepressants in resistant major depression.
 Clinical studies have shown that Lithium reduces the incidence of suicide in Bipolar disorder patients six fold seven
fold.
 This anti-suicide property is also seen in Clozapine (2nd generation anti-psychotic)

106. Mini mental status examination is:


a. Method to investigate common psychiatric problem
b. 30 point program to evaluate cognitive functions
c. To evaluate schizophrenia
d. Instrument to measure delirium

Ans: b
Ref: Kaplan and Sadock Psychiatry 11th Edition, Page No: 225
Mini-Mental State Examination (MMSE):
 The MMSE is a 30-point cognitive test developed in the mid-1970s to provide a bedside assessment of a broad
array of cognitive function, including orientation, attention, memory, construction, and language.
 It can be administered in less than 10 minutes by a busy doctor or a technician and scored rapidly by hand.
 The MMSE has been extensively studied and shows excellent reliability when raters refer to consistent scoring
rules. Validity appears good based on correlations with a wide variety of more comprehensive measures of mental
functioning and clinicopathological correlations.

107. Patients who are grandiose and require admiration from others, have which type of personality?
a. Narcissistic
b. Histrionic
c. Borderline
d. Antisocial

Ans: a
Ref: Kaplan and Sadock Psychiatry 11th Edition, Page No: 285
Personality Disorders
 Personality disorders are characterized by deeply engrained, generally lifelong maladaptive patterns of behavior
that are usually recognizable at adolescence or earlier.
 Paranoid Personality Disorder: Paranoid personality disorder is characterized by unwarranted suspicion,
hypersensitivity, jealousy, envy, rigidity, excessive self-importance, and a tendency to blame and ascribe evil
motives to others.
 Schizoid Personality Disorder: Schizoid personality disorder is characterized by shyness, oversensitivity,
seclusiveness, avoidance of close or competitive relationships, eccentricity, no loss of capacity to recognize reality,
daydreaming, and an ability to express hostility and aggression.
 Schizotypal Personality Disorder: Schizotypal personality disorder is similar to schizoid personality, but the person
also exhibits slight losses of reality testing, has odd beliefs, and is aloof and withdrawn.

www.onlinepgnepal.com Page 54
Online PG Nepal QAE Kartik 5

 Obsessive-Compulsive Personality Disorder (OCPD): OCPD is characterized by excessive concern with conformity
and standards of conscience; the individual may be rigid, overconscientious, overdutiful, overinhibited, and unable
to relax (three Ps—punctual, parsimonious, and precise).
 Histrionic Personality Disorder: Histrionic personality disorder is characterized by emotional instability,
excitability, overreactivity, vanity, immaturity, dependency, and self-dramatization that is attention seeking and
seductive.
 Avoidant Personality Disorder: Avoidant personality disorder is characterized by low levels of energy, easy
fatigability, lack of enthusiasm, inability to enjoy life, and oversensitivity to stress.
 Antisocial Personality Disorder: Antisocial personality disorder covers persons in conflict with society. They are
incapable of loyalty, selfish, callous, irresponsible, impulsive, and unable to feel guilt or learn from experience;
they have low levels of frustration tolerance and a tendency to blame others.
 Narcissistic Personality Disorder: Narcissistic personality disorder is characterized by grandiose feelings, a sense
of entitlement, a lack of empathy, envy, manipulativeness, and a need for attention and admiration.
 Borderline Personality Disorder: Borderline personality disorder is characterized by instability, impulsiveness,
chaotic sexuality, suicidal acts, self-mutilating behavior, identity problems, ambivalence, and feeling of emptiness
and boredom.
 Dependent Personality Disorder: This is characterized by passive and submissive behavior; the person is unsure of
himself or herself and becomes entirely dependent on others.
 Personality Changes Due to Another Medical Condition: This category includes alterations to a person’s
personality due to a medical condition (e.g., brain tumor).

108. False regarding anorexia nervosa:


a. Psychiatric symptoms such as depression may be associated
b. Excessive exercising can be a feature
c. Weight loss is a feature
d. Decreased appetite is a feature

Ans: d
Ref: Kaplan and Sadock Psychiatry 11th Edition, Page No: 282, 489
Feeding and Eating Disorders
 Feeding and eating disorders are characterized by a marked disturbance in eating behavior.
 Anorexia Nervosa: Anorexia nervosa is an eating disorder characterized by loss of body weight and refusal to eat.
Appetite is usually intact.
 Bulimia Nervosa: Bulimia nervosa is an eating disorder characterized by recurrent and frequent binge eating with
or without vomiting.
 Binge Eating Disorder: Binge eating disorder is a variant of bulimia nervosa with occasional, once a week, binge
eating.
 Pica: Pica is the eating of non-nutritional substances (e.g., starch).
 Rumination Disorder: The essential feature of rumination disorder is the repeated regurgitation of food, usually
beginning in infancy or childhood.

www.onlinepgnepal.com Page 55
Online PG Nepal QAE Kartik 5

Radiology
109. Which of the following is not a primary chest finding in patient with COVID19?
a. Ground-glass opacities
b. Crazy paving appearance
c. Bronchovascular thickening
d. Pleural effusion

Ans: d
Traction bronchiectasis
The primary findings on CT in adults have been reported as:
 Ground-glass
glass opacities (GGO): bilateral, subpleural, peripheral
 Crazy
razy paving appearance (GGOs and inter
inter-/intra-lobular septal thickening)
 Air space consolidation
 Bronchovascular
ronchovascular thickening in the lesion
 Traction bronchiectasis

Atypical CT findings
These findings only seen in a small minority of patients should raise concern for superadded bacterial pneumonia or other
diagnoses:
 Mediastinal lymphadenopathy
 Pleural effusions: may occur as a complication of COVID
COVID-19
 Multiple tiny pulmonary nodules (unlike many other types of viral pneumonia)
 Tree-in-bud

110. Modic changes are seen in:


a. Brain
b. Spine
c. Heart

www.onlinepgnepal.com Page 56
Online PG Nepal QAE Kartik 5

d. Lung

Ans: b
Ref: Jacob Mandell; Visual Approach to Diagnostic Imaging; Core Radiology Page No: 337
Degenerative changes in the spine are associated with vertebral body endplate and subchondral marrow signal changes
that were described and classified by Modic.

111. Which of the following cranial nerve may be seen within the carotid space?
a. CN IX
b. CN X
c. CN XI
d. All of the above

Ans: d
Ref: Jacob Mandell; Visual Approach to Diagnostic Imaging; Core Radiology Page No: 324
 The carotid space, or post-styloid parapharyngeal space, is an incomplete fascial ring surrounding the carotid
artery and jugular vein. The carotid space extends from the skull base to the aortic arch. There is some
controversy regarding the terminology of the carotid space, as some authors prefer the term “post-styloid
parapharyngeal space” versus “carotid space.” Though the former is more anatomically accurate because the
fascial boundary of this space is incomplete, the latter is simpler and perhaps more helpful to generate a
differential diagnosis of a mass.
 The contents of the carotid space include the carotid artery, carotid body, jugular vein, and several cranial nerves.
The vagus nerve (cranial nerve X) is the only cranial nerve that remains within the carotid space the entire way
into the thorax. In contrast, cranial nerves IX, XI and XII pass transiently through the carotid space. Though there
are lymph nodes surrounding the carotid space, there are no lymph nodes contained within it.
 The pattern of displacement of vascular structures in the carotid space is the key to generating a differential
diagnosis for a carotid space mass.

112. Post-traumatic saddle injury strictures are seen in:


a. Bulbous urethra
b. Penile urethra
c. Membranous urethra
d. Prostatic urethra

Ans: a
Ref: Jacob Mandell; Visual Approach to Diagnostic Imaging; Core Radiology Page No: 187
 Urethral strictures secondary to sexually transmitted disease (most commonly chronic urethritis from Neisseria
gonorrhoea) occur most commonly in the bulbous urethra. A complication of chronic urethral infection is a
periurethral abscess, which may result in a urethroperineal fistula.
 Post-traumatic saddle injury strictures also tend to occur in the bulbous urethra.
 In contrast, an iatrogenic stricture from a Foley catheter tends to occur in the penile urethra at the penile–scrotal
junction.

113. TIRADS is Imaging Reporting and Data System of:


a. Testis

www.onlinepgnepal.com Page 57
Online PG Nepal QAE Kartik 5

b. Any tumours
c. Thyroid
d. Thoracic tumours

Ans: c
ACR Thyroid Imaging Reporting and Data System (ACR TI-RADS)
ACR TI-RADS is a reporting system for thyroid nodules on ultrasound proposed by the American College of Radiology
(ACR).
This uses a standardised scoring system for reports providing users with recommendations for when to use fine needle
aspiration (FNA) or ultrasound follow-up of suspicious nodules, and when to safely leave alone nodules that are
benign/not suspicious.

114. Primary defect in penetrating atherosclerotic ulcer is seen in:


a. Intima
b. Media
c. Adventitia
d. Serosa

Ans: a
Ref: Jacob Mandell; Visual Approach to Diagnostic Imaging; Core Radiology Page No: 661
 Acute aortic syndrome represents a clinical spectrum of three related diseases that are characterized by damage
to at least one component of the aortic wall, and presents as severe chest pain.
 A defect primarily in the intima is seen in penetrating atherosclerotic ulcer (PAU).
 A defect in the media only describes intramural hematoma (IMH).
 A defect in the intima extending to media is the hallmark of aortic dissection.
 A defect in all three layers (aortic transection) is almost always due to trauma and is considered a separate entity.

115. Air bronchogram refers to:


a. Air-filled bronchi made visible by the opacified surrounding alveoli
b. Opacified bronchi visible amidst the air filled surrounding alveoli
c. Air-filled bronchi amidst the air filled alveoli
d. Opacified bronchi amidst the opacified surrounding alveoli

Ans: a
Ref: Air bronchogram by Dr Yusra Sheikh and Dr Behrang Amini et al.
Air bronchogram refers to the phenomenon of air-filled bronchi (dark) being made visible by the opacification of
surrounding alveoli (grey/white). It is almost always caused by a pathologic airspace/alveolar process, in which something
other than air fills the alveoli. Air bronchograms will not be visible if the bronchi themselves are opacified (e.g. by fluid)
and thus indicate patent proximal airways.
Air bronchograms can be seen with several processes:
 Pulmonary consolidation
 Pulmonary edema: especially with alveolar edema
 Non-obstructive atelectasis
 Severe interstitial lung disease
 Neoplasms: bronchioloalveolar carcinoma; pulmonary lymphoma

www.onlinepgnepal.com Page 58
Online PG Nepal QAE Kartik 5

 Pulmonary infarct
 Pulmonary hemorrhage
 Normal expiration

Anatomy
116. Which is not a thenar muscle?
a. Opponens pollicis
b. Opponens digiti minimi
c. Abductor pollicis brevis
d. Flexor pollicis brevis

Ans: b
Ref: USMLE Step 1 First Aid 2020
Thenar muscles are supplied by median nerve. They include:
 Opponens pollicis
 Abductor pollicis brevis
 Flexor pollicis brevis, superficial hrad (deep head is supplied by ulnar nerve)
Hypothenar muscles are supplied by ulnar nerve and they include:
 Opponens digiti minimi
 Abductor digiti minimi
 Flexor digiti minimi

117. The covering layer of nerve from outside inwards is?


a. Endoneurium, perineurium, epineurium
b. Epineurium, perineurium, endoneurium
c. Perineurium, epineurium, endoneurium
d. Perineurium, endoneurium, epineurium

Ans: b
Ref: USMLE Step 1 First Aid 2020
 Endoneurium is thin, supportive connective tissue that ensheats and supports individual myelinated nerve fibers.
 Perineurium surrounds a fascicle of nerve fibers. It is blood-nerve permeability barrier.
 Epineurium is the dense connective tissue that surrounds entire nerve
 These are arranged from outside inwards as epineurium (outer), perineurium (around) and endoneurium (inner).

118. Ovarian fossa is formed by all except:


a. Obliterated umbilical artery
b. External iliac vessels
c. Round ligament of ovary
d. Ureter

Ans: c
Ref: Gray’s anatomy, 41st Edition; Page No: 1106

www.onlinepgnepal.com Page 59
Online PG Nepal QAE Kartik 5

The ovarian fossa is a shallow depression in the peritoneal lining of the lateral pelvic wall between the peritoneal ridges
formed by the obliterated umbilical arteries anteriorly, the ureter posteriorly, and the external iliac vessels above; it lies
behind the lateral attachment of the broad ligament and usually contains the ovary in nulliparous females.

119. True about thoracic esophagus is except?


a. Lined by non-keratinized, stratified squamous epithelium.
b. Lower one third contains only smooth muscles.
c. Submucosa loosely connects the mucosa and the muscularis externa.
d. Muscularis mucosae are made up of longitudinal and circular muscle layers.

Ans: d
Ref: Gray’s anatomy, 41st Edition; Page No: 988
 Thoracic esophageal wall contains from lumen outwards, mucosa, submucosa, muscularis externa and adventitia.
 Mucosa is thick and contains epithelium, lamina propria and muscularis mucosae.
 Epithelium is non-keratinized, stratified squamous, and is continuous with that of oropharynx.
 Muscularis mucosae is composed mainly of longitudinal smooth muscle and forms a thin sheet near the
epithelium, the contours of which it follows closely.
 Submucosa loosely connects the mucosa and muscularis externa, penetrating the longitudinal ridges of the
esophageal lumen.
 Muscularis externa consists of outer longitudinal and inner circular layers.
 Adventitia is not well defined.

120. Blood supply of middle third part of esophagus:


a. Aorta
b. Inferior thyroid artery
c. Gastric artery
d. Superior thyroid artery

Ans: a
Ref: Snell Clinical Anatomy; 9th edition; Page No: 100
Esophagus
• The esophagus is a tubular structure about 10 in. (25 cm) long that is continuous above with the laryngeal part of
the pharynx opposite the sixth cervical vertebra. It passes through the diaphragm at the level of the 10th thoracic
vertebra to join the stomach. In the neck, the esophagus lies in front of the vertebral column; laterally, it is related
to the lobes of the thyroid gland; and anteriorly, it is in contact with the trachea and the recurrent laryngeal
nerves in the thorax, it passes downward and to the left through the superior and then the posterior
mediastinum. At the level of the sternal angle, the aortic arch pushes the esophagus over to the midline.
• In the abdomen, the esophagus descends for about 0.5 in. (1.3 cm) and then enters the stomach. It is related to
the left lobe of the liver anteriorly and to the left crus of the diaphragm posteriorly.
Blood Supply of the Esophagus
• The upper third of the esophagus is supplied by the inferior thyroid artery, the middle third by branches from
the descending thoracic aorta, and the lower third by branches from the left gastric artery. The veins from the
upper third drain into the inferior thyroid veins, from the middle third into the azygos veins, and from the lower
third into the left gastric vein, a tributary of the portal vein.
Lymph Drainage of the Esophagus

www.onlinepgnepal.com Page 60
Online PG Nepal QAE Kartik 5

• Lymph vessels from the upper third of the esophagus drain into the deep cervical nodes, from the middle third
into the superior and posterior mediastinal nodes, and from the lower third into nodes along the left gastric blood
vessels and the celiac nodes.
Nerve Supply of the Esophagus
• The esophagus is supplied by parasympathetic and sympathetic efferent and afferent fibers via the vagi and
sympathetic trunks. In the lower part of its thoracic course, the esophagus is surrounded by the esophageal nerve
plexus.

121. The content of deep perineal pouch are all except:


a. Root of penis
b. Membranous part of the urethra
c. Sphincter urethrae
d. Bulbourethral gland

Ans: a
Ref: Snell Clinical Anatomy; 9th edition; Page No: 319
Contents of the Superficial Perineal Pouch in the Male
 The superficial perineal pouch contains structures forming the root of the penis, together with the muscles that
cover them—namely, the bulbospongiosus muscles and the ischiocavernosus muscles. The bulbospongiosus
muscles situated one on each side of the midline cover the bulb of the penis and the posterior portion of the
corpus spongiosum.
 Their function is to compress the penile part of the urethra and empty it of residual urine or semen. The anterior
fibers also compress the deep dorsal vein of the penis, thus impeding the venous drainage of the erectile tissue
and thereby assisting in the process of erection of the penis.
Contents of the Deep Perineal Pouch in the Male
• The deep perineal pouch contains the membranous part of the urethra, the sphincter urethrae, the
bulbourethral glands, the deep transverse perineal muscles, the internal pudendal vessels and their branches,
and the dorsal nerves of the penis.

122. Lining epithelium of vagina is:


a. Pseudostratified columnar epithelium
b. Keratinized stratified squamous epithelium
c. Non keratinized stratified squamous epithelium
d. Ciliated columnar epithelium

Ans: c
Ref: Gray’s anatomy, 41st Edition; Page No: 1291
Vagina
• The vagina is a fibromuscular tube lined by non-keratinized stratified epithelium. It extends from the vestibule
(the opening between the labia minora) to the uterus. The upper end of the vagina surrounds the vaginal
projection of the uterine cervix. The anular recess between the cervix and vagina is the fornix; the different parts
of the fornix are given separate names, i.e. anterior, posterior and right and left lateral, but they are continuous.
• The apex of the vagina is directed posteriorly towards the ischial spines. The width of the vagina increases as it
ascends. The inner surfaces of the anterior and posterior vaginal walls are in contact with each other, forming a

www.onlinepgnepal.com Page 61
Online PG Nepal QAE Kartik 5

transverse slit. The vaginal mucosa is attached to the uterine cervix higher on the posterior cervical wall than on
the anterior; the anterior wall is approximately 7.5 cm long and the posterior wall is approximately 9 cm long.
• The fibromuscular anterior wall of the vagina supports the base of the bladder in its middle and upper portions,
and the urethra (which is embedded in it) inferiorly. The fibromuscular posterior wall of the vagina supports the
rectum.
• The upper quarter of the posterior vagina is separated from the rectum by the peritoneum of the recto-uterine
pouch, and by moderately dense fibromuscular tissue (Denonvilliers’s fascia) in its middle half.
• In its lower quarter, it is separated from the anal canal by the fibromuscular perineal body.
• The upper part of the vagina is supported laterally by levator ani, together with the transverse cervical,
pubocervical and uterosacral ligaments. Pubovaginalis provides a U-shaped muscular sling around the mid-vagina.
• The lower vagina is surrounded by the skeletal muscle fibres of bulbospongiosus. As the ureters pass
anteromedially to reach the fundus of the bladder, they pass close to the lateral Fornices, where care must be
taken to avoid damage during hysterectomy. As they enter the bladder, the ureters are usually anterior to the
vagina; at this point, each ureter is crossed transversely by a uterine artery.

123. Tonsils develop embryologically from:


a. First pharyngeal pouch
b. Second pharyngeal pouch
c. Third pharyngeal pouch
d. Fourth pharyngeal pouch

Ans: b
Ref: IB Singh’s Human Embryology, 8th Edition; Page No: 110
FATE OF ENDODERMAL POUCHES
The endodermal pouches take part in the formation of several important organs.
These are listed below:
First Pouch
• Its ventral part is obliterated by formation of the tongue.
• Its dorsal part receives a contribution from the dorsal part of the second pouch, and these two together forms a
diverticulum that grows towards the region of the developing ear. This diverticulum is called the tubotympanic
recess. The proximal part of this recess gives rise to the auditory (pharyngotympanic) tube and the distal part to
the middle ear cavity, including the tympanic antrum.
Second Pouch
• The epithelium of the ventral part of this pouch contributes to the formation of the tonsil.
• The dorsal part takes part in the formation of the tubotympanic recess.

Third Pouch
• This gives rise to the inferior parathyroid glands, and the thymus.
Fourth Pouch
• This gives origin to the superior parathyroid glands, and may contribute to the thyroid gland.
Fifth or Ultimobranchial Pouch
• A fifth pouch is seen for a brief period during development. In some species it gives rise to the ultimobranchial
body. Its fate in man is controversial. It is generally believed to be incorporated into the fourth pouch, the two
together forming the caudal pharyngeal complex. The superior parathyroid glands arise from this complex. The
complex probably also gives origin to the parafollicular cells of the thyroid gland.

www.onlinepgnepal.com Page 62
Online PG Nepal QAE Kartik 5

124. The posterior wall of Axilla is bounded by the following structures, except:
a. Subscapularis
b. Pectoralis major
c. Teres major
d. Latissimus dorsi

Ans: b
Ref: Moore's Clinically Oriented Anatomy; 7th Edition, Page No: 713
The axilla has an apex, a base, and four walls (three of which are muscular):
 The apex of axilla is the cervico-axillary canal, the passageway between the neck and axilla, bounded by the 1st
rib, clavicle, and superior edge of the scapula. The arteries, veins, lymphatics, and nerves traverse this superior
opening of the axilla to pass to or from the arm.
 The base of axilla is formed by the concave skin, subcutaneous tissue, and axillary (deep) fascia extending from the
arm to the thoracic wall (approximately the 4th rib level), forming the axillary fossa (armpit). The base of the axilla
and axillary fossa are bounded by the anterior and posterior axillary folds, the thoracic wall, and the medial aspect
of the arm.
 The anterior wall of axilla has two layers, formed by the pectoralis major and pectoralis minor and the pectoral
and clavicopectoral fascia associated with them. The anterior axillary fold is the inferior most part of the anterior
wall that may be grasped between the fingers; it is formed by the pectoralis major, as it bridges from thoracic wall
to humerus, and the overlying integument
 The posterior wall of axilla is formed chiefly by the scapula and subscapularis on its anterior surface and inferiorly
by the teres major and latissimus dorsi. The posterior axillary fold is the inferior most part of the posterior wall
that may be grasped. It extends farther inferiorly than the anterior wall and is formed by latissimus dorsi, teres
major and overlying integument.

125. What is the site of origin of third order neurons in spino-thalamic tract?
a. Cerebral cortex
b. Nucleus of thalamus
c. Peripheral sensory receptors
d. Posterior horn gray matter

Ans: b
Ref: Gray’s Anatomy for Students, 4th edition, e41-42
 The first-order neuronal cell body of axons forming the spinothalamic tract is located in a spinal ganglion. Axons
then enter the spinal cord through the posterior root to reach the posterior horn.
 From here, axons have two courses: some synapse immediately on second-order neurons in the posterior horn
gray matter (lamina I and V), and others have axonal collaterals that ascend or descend one to two spinal cord
segments in the posterolateral tract of Lissauer before synapsing with the second-order neurons in the gray
matter.
 Axons of the second-order neurons then cross obliquely over two to three spinal cord segments within the
anterior commissure of the spinal cord to join the anterolateral tract on the contralateral side.
 These second-order axons ascend through the CNS to reach the third-order neuronal cell bodies in the ventral
posterior lateral nucleus of the thalamus.

www.onlinepgnepal.com Page 63
Online PG Nepal QAE Kartik 5

 Axons from the third-order


order neurons tthen
hen project through the posterior limb of the internal capsule to reach the
primary somatosensory cortex.

126. Segment II of right lung is:


a. Apical segment
b. Anterior segment
c. Medial segment
d. Posterior segment

Ans: d
Ref: Gray’s Anatomy for Students, 4th edition,
ition, 176
A bronchopulmonary segment is the area of lung supplied by a segmental bronchus and its accompanying pulmonary
artery branch. Segment II is posterior segment.

www.onlinepgnepal.com Page 64
Online PG Nepal QAE Kartik 5

127. What is the origin of greater pancreatic artery?


a. Gastroduodenal artery
b. Splenic artery
c. Superior mesenteric artery
d. None

Ans: b
Ref: Gray’s Anatomy for Students, 4th edition, 335
The arterial supply to the pancreas includes the:
 gastroduodenal artery from the common hepatic artery (a branch of the celiac trunk),

www.onlinepgnepal.com Page 65
Online PG Nepal QAE Kartik 5

 anterior superior pancreaticoduodenal


reaticoduodenal artery from the gastroduodenal artery,
 posterior superior pancreaticoduodenal artery from the gastroduodenal artery,
 dorsal pancreatic artery from the inferior pancreatic artery (a branch of the splenic artery),
 greater pancreatic artery from
om the inferior pancreatic artery (a branch of the splenic artery),
 anterior inferior pancreaticoduodenal artery from the inferior pancreaticoduodenal artery (a branch of the
superior mesenteric artery), and
 posterior inferior pancreaticoduodenal artery fr from
om the inferior pancreaticoduodenal artery (a branch of the
superior mesenteric artery)

Physiology
128. RMP of neuron is:
a. -60 mV
b. +60 mV
c. -70 mV
d. +70 mV

Ans: c
Ref: Berne and Levy Physiology; 7th edition; Pg 65
When a microelectrode (tip diameter <0.5 µm) is inserted through the plasma membrane of a neuron, a difference in
potential is observed between the tip of the microelectrode inside the cell and an electrode placed outside the cell. The
internal electrode is approximately 70 mV negative with rega
regard
rd to the external electrode, and this difference is referred to
as the resting membrane potential or, simply, the resting potential. (By convention, membrane potentials are expressed as
the intracellular potential minus the extracellular potential.) Neuronsns have a resting potential that typically is around −70
mV.

www.onlinepgnepal.com Page 66
Online PG Nepal QAE Kartik 5

129. Ruffini end organ is associated with sensation of:


a. Cold
b. Vibration
c. Position sense
d. Pressure

Ans: d
Ref: Ganong Review of Medical Physiology, Lange; 25th Edition, Page No: 160
Touch and pressure are sensed by four types of mechanoreceptors.
• Meissner corpuscles are dendrites encapsulated in connective tissue and respond to changes in texture and slow
vibrations.
• Merkel cells are expanded dendritic endings, and they respond to sustained pressure and touch.
• Ruffini corpuscles are enlarged dendritic endings with elongated capsules, and they respond to sustained
pressure.
• Pacinian corpuscles consist of unmyelinated dendritic endings of a sensory nerve fiber, 2 μm in diameter,
encapsulated by concentric lamellae of connective tissue that give the organ the appearance of a cocktail onion.
These receptors respond to deep pressure and fast vibration.
• The sensory nerves from these mechanoreceptors are large myelinated Aα and Aβ fibers whose conduction
velocities range from ~70–120 to ~40–75 m/s, respectively.

130. Nitrogen washout method is useful for estimating:


a. Dead space volume
b. Tidal volume
c. Diffusion capacity
d. Functional residual capacity

Ans: d
Ref: Understanding medical physiology Bijalani; 4th edition; Pg 252
Functional Residual Capacity
• An increase in functional residual capacity (FRC) takes place through air trapping in diseases associated with
chronic increase in airway resistance. FRC cannot be measured by spirometry. It may be measured either by the
helium dilution method or the nitrogen washout method.

Nitrogen Washout Method


• This method is based on the knowledge that the alveolar nitrogen concentration is normally about 80%. If the
subject washes out the nitrogen in the lungs completely, then from the amount of nitrogen washed out and the
knowledge that it formed 80% of the air in the lung, the volume of air in the lungs can be determined.
• Starting with the normal end-expiratory position, the subject breathes 100% oxygen for a few minutes to wash out
the nitrogen. In a normal adult, about 2 min of breathing nitrogen free air is enough to wash out all the nitrogen.
However, in patients with poorly ventilated sections within the lungs, upto 7 min may be required for the purpose.
The expired air is collected in a big spirometer. The volume of air collected in the spirometer is noted and its
nitrogen concentration determined.

131. Greatest area is covered by:

www.onlinepgnepal.com Page 67
Online PG Nepal QAE Kartik 5

a. Arterioles
b. Venules
c. Capillaries
d. Veins

Ans: c
Ref: Ganong Review of Medical Physiology, Lange; 25th Edition, Page No: 568
The approximate total cross sectional area of different blood vessels are:
 Aorta: 4.5 cm2
 Artery: 20 cm2
 Arteriole: 400 cm2
 Capillary: 4500 cm2
 Venule: 4000 cm2
 Vein: 40 cm2
 Venacava: 18 cm2

132. Anterolateral system carries all except:


a. Pain
b. Sexual sensation
c. Temperature
d. Vibration

Ans: d
Ref: Guyton and Hall Textbook of Medical Physiology, Elsevier; 13th Edition, Page No: 609
Anterolateral system carries:
 Pain
 Thermal sensations including both warm and cold sensations
 Crude touch and pressure sensations capable only of crude localization
 Tickle and itch sensation
 Sexual sensation

Dorsal column carries:


 Touch sensation requiring a high degree of localization of stimulus
 Touch sensation requiring transmission of graded intensity
 Phasic sensation such as vibration
 Sensations that signal movement against the skin
 Position sensation from joints
 Pressure sensation related to fine degrees of judgment of pressure intensity

133. Spermatozoa’s fructose is transported by:


a. GLUT 3
b. GLUT 4
c. GLUT 5
d. GLUT 7

www.onlinepgnepal.com Page 68
Online PG Nepal QAE Kartik 5

Ans: c
Ref: Ganong Review of Medical Physiology, Lange; 25th Edition, Page No: 433

134. Total blood volume iss what percentage of body weight?


a. 4%
b. 5%
c. 8%
d. 10%

Ans: c
Ref: Ganong Review of Medical Physiology, Lange; 25th Edition, Page No: 33-4
 In the average young adult male, 18% of the body weight is protein and related substances, 7% is mineral, and
15% is fat. The remaining 60% is water.
 The intracellular component of water accounts for about 40% of body weight and extracellular component for
about 20%.
 Approximately 25% of the extracellular component is in the vascular system (plasma = 5% of body weight) and
75% outside the blood vessels (interstitial fluid = 15% of body weight).
 Total blood volume is about 8% of body weight.

135. Maximum absorption of short chain fatty acid produced by bacteria occurs in?
a. Duodenum
b. Jejunum
c. Ileum
d. Colon

Ans: d

www.onlinepgnepal.com Page 69
Online PG Nepal QAE Kartik 5

Ref: Ganong Review of Medical Physiology, Lange; 25th Edition, Page No: 483, 493
Short chain fatty acids (SCFAs) are 2 – 5 carbon weak acids that have an average normal concentration of about 80 mmol/L
in the lumen. They are formed by the action of colonic bacteria on complex carbohydrates, resistant starches, and other
components of the dietary fiber, that is, the material that escapes digestion in the upper gastrointestinal tract and enters
colon. SCFAs that are produced in the colon (by bacterial action) are absorbed from it.

136. Glucose dependent insulinotropic peptide is secreted from?


a. G cells
b. I cells
c. K cells
d. S cells

Ans: c
Ref: USMLE Step 1 First Aid 2020
 Glucose dependent insulinotropic peptide is also known as gastric inhibitory peptide (GIP).
 It is secreted from K cells of duodenum and jejunum. Secretion is increased by fatty acids, amino acids and oral
glucose.
 It causes decreased gastric acid secretion and increased insulin release.
 Oral glucose load leads to increased insulin as compared to intravenous glucose due to GIP.

137. Which ion channel opens during Phase 1 of myocardial action potential?
a. Sodium
b. Chloride
c. Potassium
d. Calcium

Ans: c
Ref: USMLE Step 1 First Aid 2020
The following changes are seen in myocardial action potential
Phase Features
0 Rapid upstroke and depolarization – voltage gated sodium channels open
1 Initial repolarization – inactivation of voltage gated sodium channels while
voltage gated potassium channels begin to open
2 Plateau phase – calcium influx through voltage gated sodium channel
balances potassium efflux; calcium influx triggers further release of calcium
from sarcoplasmic reticulum and myocyte contraction.
3 Rapid repolarization – massive potassium efflux due to opening of voltage
gated slow potassium channels and closure of voltage gated calcium
channels.
4 Resting potential – high potassium permeability through potassium channels.

138. Which is factor VII?


a. Proaccelerin
b. Proconvertin
c. Stuart-Prower factor
d. Hageman factor

www.onlinepgnepal.com Page 70
Online PG Nepal QAE Kartik 5

Ans: b
Ref: Ganong Review of Medical Physiology, Lan
Lange; 25th Edition, Page No: 562

139. Responses to high altitude are all except:


a. Increased PaCO2
b. Decreased PaO2
c. Increased 2,3 – BPG
d. Increased number of mitochondria per cell

Ans: a
Ref: USMLE Step 1 First Aid 2020
Responses to high altitude are:
 Decreased PiO2 causing decreased PaO2 which triggers increased ventilation causing decreased PaCO2 thereby
resulting in respiratory alkalosis and altitude sickness.
 Chronic increase in ventilation.
 Increased erythropoietin resulting in increased hemoglobin and h hematocrit.
 Increased 2,3 – BPG which causes right shift of oxy oxy-hemoglobin
hemoglobin dissociation curve and thus releasing more oxygen
to tissue.
 Increased number of mitochondria inside cells
 Increased renal excretion of bicarbonate to compensate for respiratory alkaalkalosis
 Chronic hypoxic pulmonary vasoconstriction results in pulmonary hypertension and right ventricular hypertrophy.

www.onlinepgnepal.com Page 71
Online PG Nepal QAE Kartik 5

Pathology
140. Which of the following is associated with intrinsic pathway of apoptosis?
a. Activation of caspase 8
b. Activation of caspase 9
c. Fas ligand binding to its receptor
d. Activation of TNF receptor

Ans: b
Ref: Robbins Basic Pathology 10th Edition, Page No: 39
All of the options given are of extrinsic or death receptor pathway of apoptosis except option b
(iNtrinsic=Nine caspase-9
Extrinsic=Eightcaspase-8)

141. Not true among the following is:


a. Most bacterial infections are associated with neutrophilia
b. Infectious mononucleosis is associated with lymphopenia
c. Allergies are associated with eosinophilia
d. Typhoid fever is associated with leucopenia

Ans: b
Ref: Robbins Basic Pathology 10th Edition, Page No: 87
Most bacterial infections induce an increase in the blood neutrophil count, called neutrophilia. Viral infections, such as
infectious mononucleosis, mumps, and German measles, cause an absolute increase in the number of lymphocytes
(lymphocytosis). In some allergies and parasitic infestations, there is an increase in the number of blood eosinophils,
creating an eosinophilia.
Certain infections (typhoid fever and infections caused by some viruses, rickettsiae, and certain protozoa) are associated
with a decreased number of circulating white cells (leukopenia).

142. Which of the following manifestations of SLE is less common among the given options?
a. Anemia
b. Arthralgia
c. Skin
d. Raynaud phenomenon

Ans: d
Ref: Robbins Basic Pathology 10th Edition, Page No: 154

www.onlinepgnepal.com Page 72
Online PG Nepal QAE Kartik 5

143. Choose the correct pair


a. Bloom syndrome: Defects in the mismatch repair system
b. Xeroderma pigmentosum: Defect in the nucleotide excision repair pathway
c. HNPCC syndrome: Defects in homologo
homologous recombination DNA repair system
d. All are correct

Ans: b
Ref: Robbins Basic Pathology 10th Edition, Page No: 228
 Patients with HNPCC syndrome have defects in the mismatch repair system, leading to
development of carcinomas of the colon. These patients’ genomes show microsatellite instability
(MSI), characterized by changes in length of short tandem repeating sequences throughout the
genome.
 Patients with xeroderma pigmentosum have a defect in the nucleotide excision repair pathway.
They are at increased risk for the development of skin cancers in sites exposed to sun-light
because of an inability to repair pyrimidine dimers induced by UV light.
 Syndromes involving defects in the homologous recombination DNA repair system constitute a
group of disorders—Bloom
Bloom syndrome, ataxia-telangiectasia, and Fanconi anemia—that are
characterized by hypersensitivity to DNA-damaging agents, such as ionizing radiation. BRCA1 and
BRCA2, which are mutated in familial breast cancers, also are involved in homologous DNA
repair.
 Mutations incurred in lymphocytes expressing gene products that induce genomic instability
(RAG1, RAG2, AID) are important in the pathogenesis of lymphoid neoplasms.

144. Aging pigment or wear and tear pigment is known as:


a. Hemosiderin
b. Lipofuscin
c. Cytochrome
d. Carotenoids

www.onlinepgnepal.com Page 73
Online PG Nepal QAE Kartik 5

Ans: b
Ref: Robbins Basic Pathology 9th Edition, Page No: 64
• Lipofuscin is an insoluble pigment, also known as lipochrome or wear-and-tear pigment.
• Lipofuscin is composed of polymers of lipids and phospholipids in complex with protein, suggesting that it is
derived through lipid peroxidation of polyunsaturated lipids of subcellular membranes
• Lipofuscin is not injurious to the cell or its functions. Its importance lies in its being a telltale sign of free radical
injury and lipid peroxidation. The term is derived from the Latin ( fuscus, brown), referring to brown lipid.
• In tissue sections it appears as a yellow-brown, finely granular cytoplasmic, often perinuclear, pigment.
• It is seen in cells undergoing slow, regressive changes and is particularly prominent in the liver and heart of aging
patients or patients with severe malnutrition and cancer cachexia.

145. White infarcts are seen in all except:


a. Lung
b. Kidney
c. Spleen
d. Heart

Ans: a
Ref: Robbins Basic Pathology 9th Edition, Page No: 129-130
Infarcts are classified according to color and the presence or absence of infection; they are either red (hemorrhagic) or
white (anemic) and may be septic or bland.
Red infarcts occur:
1. With venous occlusions(e.g., testicular torsion
2. In loose, spongy tissues (e.g., lung) where blood can collect in the infarcted zone,
3. In tissues with dual circulations (e.g., lung and small intestine) that allow blood to flow from an unobstructed
parallel supply into a necrotic zone,
4. In tissues previously congested by sluggish venous outflow, and
5. When flow is reestablished to a site of previous arterial occlusion and necrosis (e.g., following angioplasty of an
arterial obstruction).
White infarcts: occur with arterial occlusions in-solid organs with end-arterial circulation (e.g., heart, spleen, and
kidney), and where tissue density limits the seepage of blood from adjoining capillary beds into the necrotic area.

146. The term ragged red fibers is applied to describe the skeletal muscle fibers in:
a. DMD
b. Myotonic dystrophy
c. Amyotrophic lateral sclerosis
d. Mitochondrial myopathy

Ans: d
Ref: Robbins Basic Pathology 9th Edition, Page No: 1245
Mitochondrial Myopathies
 Mitochondrial diseases are complex systemic conditions that can involve many organ systems, including skeletal
muscle. The genetics of these disorders are varied and unusually complex, but many of the causative mutations
appear to impair the ability of mitochondria to generate ATP.

www.onlinepgnepal.com Page 74
Online PG Nepal QAE Kartik 5

As a result, these diseases tend to affect skeletal muscles and other tissues rich in cell types with high ATP
requirements, particularly cardiac muscle cells and neurons. Skeletal muscle involvement can manifest as
weakness, elevations in serum creatine kinase levels, or rhabdomyolysis.
 Although the anatomic pattern of muscle weakness is variable, involvement of extraocular eye muscles is common
and can be a clue to the diagnosis.
 Indeed, chronic progressive external ophthalmoplegia is a common feature of mitochondrial disorders, and may
occur as an isolated phenomenon or a part of a multisystem syndrome. The reason that extraocular eye muscles
are particularly sensitive to mitochondrial disease is uncertain, but it may be that these muscles have
exceptionally high requirements for ATP. In line with this idea, extraocular eye muscles have the most
mitochondria per mass of any of the body’s muscles.
MORPHOLOGY
• The most consistent pathologic change in skeletal muscle is abnormal aggregates of mitochondria that are seen
preferentially in the subsarcolemmal area of affected myofibers, producing an appearance that is referred to as
“ragged red fibers”.
• By electron microscopy, morphologically abnormal mitochondria are seen. Loss of particular mitochondrial
enzyme activities characterizes some mitochondrial diseases and may be appreciated by histochemical staining for
cytochrome oxidase.
• Some mitochondrial diseases lack morphologic changes and can only be diagnosed through enzymatic assays or
genetic analyses.

147. Posttranscriptional silencing of gene expression is done by


a. miRNA
b. siRNA
c. IncRNA
d. CRISPR associated genes

Ans: a
Ref: Robbins Basic Pathology 10th Edition, Page No: 4-5
 Micro-RNAs (miRNAs) are relatively short RNAs (22 nucleotides on average) that function primarily to modulate
the translation of target mRNAs into their corresponding proteins. Posttranscriptional silencing of gene expression
by miRNA is a fundamental and evolutionarily conserved mechanism of gene regulation present in all eukaryotes.
 Small interfering RNAs (siRNAs) are short RNA sequences. Synthetic siRNAs that can target specific mRNA species
are therefore powerful laboratory tools to study gene function (so-called knockdown technology); they also are
promising as therapeutic agents to silence pathogenic genes, e.g., oncogenes involved in neoplastic
transformation.
 lncRNAs(Long noncoding RNA) modulate gene expression in many ways (Fig. 1.4); for example, they can bind to
regions of chromatin, restricting RNA polymerase access to coding genes within the region.
 Exciting new developments that permit exquisitely specific genome editing stand to usher in an era of molecular
revolution. These advances come from a wholly unexpected source: the discovery of clustered regularly
interspaced short palindromic repeats (CRISPRs) and Cas (or CRISPRassociated genes).

148. Which of the following intermediate filament is produced by glial cells that support neurons?
a. Neurofilament
b. Microtubules
c. Cytokeratin

www.onlinepgnepal.com Page 75
Online PG Nepal QAE Kartik 5

d. GFAP

Ans: d
Ref: Robbins Basic Pathology 10th Edition, Page No: 11
Intermediate filaments are fibrils 10-nm in diameter that comprise a large and heterogeneous family. Members include
lamins A, B, and C, which contribute to the structure of nuclear lamina. Individual types of intermediate filaments have
characteristic tissue-specific patterns of expression that are useful for identifying the cellular origin of poorly
differentiated tumors.
 Vimentin: Mesenchymal cells (fibroblasts, endothelium) anchoring intracellular organelles
 Desmin: Muscle cells, forming the scaffold on which actin and myosin contract • Neurofilaments: Axons of
neurons, imparting strength and rigidity
 Glial fibrillary acidic protein(GFAP): Glial cells that support neurons
 Cytokeratins: Epithelial cells express more than 30 distinct varieties with distinct patterns of expression in
different lineages (e.g., lung versus gastrointestinal epithelia). These can serve as histochemical markers for
various epithelia

149. Which of the following cell junction mechanically attach cells—and their intracellular cytoskeletons—to other cells
or to the ECM?
a. Desmosomes
b. Occluding junctions
c. Gap junctions
d. Tight junctions

Ans: a
Ref: Robbins Basic Pathology 10th Edition, Page No: 12
Cell junctions are organized into three basic types
 Occluding junctions (tight junctions) seal adjacent cells together to create a continuous barrier that restricts the
paracellular (between cells) movement of ions and other molecules.
 Anchoring junctions (desmosomes) mechanically attach cells—and their intracellular cytoskeletons—to other cells
or to the ECM.
 Communicating junctions (gap junctions) mediate the passage of chemical or electrical signals from one cell to
another.

Pharmacology
150. Which of the following is not an example of phase II drug reaction?
a. Methylation
b. Acetylation
c. Decyclization
d. Glucoronidation

Ans: c
Ref: KD Tripathi Essentials of Medical Pharmacology; 7th Edition, Page No: 23-24
(a) Nonsynthetic/Phase I/Functionalization reactions: a functional group is generated or exposed— metabolite may
be active or inactive.

www.onlinepgnepal.com Page 76
Online PG Nepal QAE Kartik 5

(b) Synthetic/Conjugation/ Phase II reactions: an endogenous radical is conjugated to the drug— metabolite is mostly
inactive; except few drugs, e.g. glucuronide conjugate of morphine and sulfate conjugate of minoxidil are active.

Nonsynthetic reactions
• Oxidation
• Reduction
• Hydrolysis
• Cyclization
• Decyclization

Synthetic reactions
• Glucuronide conjugation
• Acetylation
• Methylation
• Sulfate conjugation
• Glycine conjugation
• Glutathione conjugation
• Ribonucleoside/nucleotide synthesis

151. Potency of drug is measure of its:


a. Safety
b. Efficacy
c. Therapeutic index
d. Dose

Ans: d
Ref: KD Tripathi Essentials of Medical Pharmacology; 7th Edition, Page No: 54
• The position of DRC on the dose axis is the index of drug potency which refers to the amount of drug needed to
produce a certain response.
• Thus, if 10 mg of morphine = 100 mg of pethidine as analgesic, morphine is 10 times more potent than pethidine.
However, a higher potency, in itself, does not confer clinical superiority unless the potency for therapeutic effect is
selectively increased over potency for adverse effect. Drug potency is clearly a factor in choosing the dose of a
drug.
• The upper limit of DRC is the index of drug efficacy and refers to the maximal response that can be elicited by the
drug, e.g. morphine produces a degree of analgesia not obtainable with any dose of aspirin—morphine is more
efficacious than aspirin. Efficacy is a more decisive factor in the choice of a drug.

152. Cleft lip is caused by which of the following drug:


a. Phenobarbitone
b. Phenytoin
c. Sodium valproate
d. Levatiracetam

Ans: b
Ref: KD Tripathi Essentials of Medical Pharmacology; 7th Edition, Page No: 414, 417

www.onlinepgnepal.com Page 77
Online PG Nepal QAE Kartik 5

Phenytoin
Adverse effects
After prolonged use numerous side effects are produced at therapeutic plasma concentration; others occur as a
manifestation of toxicity due to overdose.
At therapeutic levels
• Gum hypertrophy: Commonest (20% incidence), more in younger patients. It is due to overgrowth of gingival
collagen fibres. This can be minimized by maintaining oral hygiene. • Hirsutism, coarsening of facial features
(troublesome in young girls), acne.
• Hypersensitivity reactions are—rashes, DLE, lymphadenopathy; neutropenia is rare but requires discontinuation of
therapy.
• Megaloblastic anaemia: Phenytoin decreases folate absorption and increases its excretion. • Osteomalacia:
Phenytoin interferes with metabolic activation of vit D and with calcium absorption/metabolism.
• It can inhibit insulin release and cause hyperglycaemia.
• Used during pregnancy, phenytoin can produce ‘foetal hydantoin syndrome’ (hypoplastic phalanges, cleft
palate, hare lip, microcephaly), which is probably caused by its areneoxide metabolite.

Valproic acid (Sodium valproate)


Adverse effects
The toxicity of valproate is relatively low.
• Anorexia, vomiting, loose motions and heart burn are common but mild. Drowsiness, ataxia and tremor are
dose-related side effects. However, cognitive and behavioral effects are not prominent.
• Alopecia, curling of hair, weight gain and increased bleeding tendency have been observed. Rashes and
thrombocytopenia are infrequent hypersensitivity phenomena.
• Asymptomatic rise in serum transaminase is often noted; monitoring of liver function is advised.
• Used during pregnancy, it has produced spina bifida and other neural tube defects in the offspring; should
be avoided.

153. Drug used in benign prostatic hypertrophy:


a. Alpha 1 agonist
b. Alpha 1 antagonist
c. Alpha 2 agonist
d. Alpha 2 antagonist

Ans: b
Ref: KD Tripathi Essentials of Medical Pharmacology; 7th Edition, Page No: 140
Tone of smooth muscle in bladder trigone, sphincter and prostate is reduced by blockade of α1 receptors (mostly of the
α1A subtype) → urine flow in panents with benign hypertrophy of prostate (BHP) is improved.

154. Which is not a short acting beta-agonist?


a. Albuterol
b. Bambuterol
c. Pirbuterol
d. Terbutaline

Ans: b

www.onlinepgnepal.com Page 78
Online PG Nepal QAE Kartik 5

Ref: Katzung’s Basic & Clinical Pharmacology; Lange; 14th Edition, Page No: 351
 Ultra long acting Beta agonists need to be taken only once a day but because of their prolonged bronchodilation
masks symptoms of bronchial inflammation, they should be used only with inhaled corticosteroid for asthma but
can be used as monotherapy for COPD. Eg: indacaterol, oledaterol, vilanterol, and bambuterol.
 Salmeterol and formeterol are long acting beta-2 selective agonists with 12 hour duration of action while
albuterol, terbutaline, metaproterenol, and pirbuterol are short acting beta-2 agonists.

155. Drug with both antidepressant and antipsychotic property is:


a. Mianserin
b. Amoxapine
c. Tianeptine
d. Mirtazapine

Ans: b
Ref: KD Tripathi Essentials of Medical Pharmacology; 8th Edition, Page No: 491
 Amoxapine is a tetracyclic compound and is unusual in that it blocks dopamine D2 receptors in addition to
inhibiting noradrenaline uptake. It has mixed antidepressant and neuroleptic properties. It may be used in
patients with psychotic depression unresponsive to other antidepressants.
 Mianserin is unique in not inhibiting either NA or 5-HT uptake; but blocks pre-synaptic α2 receptors thereby
increasing release and turnover of NA in brain which may be responsible for its antidepressant effect.
 Tianeptine increases rather than inhibits 5-HT uptake, and is neither sedative nor stimulant. It is efficacious in
anxiodepressive states as well as in endogenous depression.
 Miratazapin acts by novel antidepressant mechanism, it blocks α2auto- (on NA neurons) and hetero- (on 5-HT
neurons) receptors enhancing both NA and 5-HT release

156. What is ddC also known as?


a. Didanosine
b. Lamivudine
c. Stavudine
d. Zalcitabine

Ans: d
Ref: Katzung’s Basic & Clinical Pharmacology; Lange; 14th Edition, Page No: 864

www.onlinepgnepal.com Page 79
Online PG Nepal QAE Kartik 5

157. Omeprazole is an inhibitor of and hence should not be given with clopidogrel:
a. CYP2C19
b. CYP2C9
c. CYP1A1
d. CYP2E1

Ans: a
Ref: KD Tripathi Essentials of Medical Pharmacology; 7th Edition, Page No: 23, 630
CYP2C19 Metabolizes > 12 frequently used drugs including omeprazole, lansoprazole, phenytoin, diazepam, propranolol.
Rifampicin and carbamazepine are potent inducers of the CYP2C subfamily, while omeprazole is an inhibitor.
inhibitor
Omeprazole, an inhibitor of CYP2C19, reduces metabolic activation of clopidogrel and its antiplatelet action.

158. Which of the following is used in conjunction with a Thiazide type or a high ceiling diuretic to prevent hypokalemia
and slightly augment the natriuretic response
a. Acetazolamide
b. Mannitol
c. Triamterene
d. Chlorpropamide

Ans: c
Ref: KD Tripathi Essentials of Medical Pharmacology; 7th Edition, Page No: 590

www.onlinepgnepal.com Page 80
Online PG Nepal QAE Kartik 5

Both triamterene and amiloride are used in conjunction with a thiazide type or a high ceiling diuretic to prevent
hypokalaemia and slightly augment the natriuretic response. The antihypertensive action of thiazide is also supplemented.
Risk of hyperkalaemia is the most important adverse effect of amiloride and triamterene. These drugs should not be given
with K + supplements; dangerous hyperkalaemia may develop. Hyperkalaemia is also more likely in patients receiving ACE
inhibitors/ARBs, beta blockers, NSAIDs and in those with renal impairment.
Both drugs elevate plasma digoxin levels.

159. Buprenorphine has been used in all of the following except:


a. Cancer pain
b. Premedication
c. Myocardial infarction
d. Analgesia during labour

Ans: d
Ref: KD Tripathi Essentials of Medical Pharmacology; 7th Edition, Page No: 482
Buprenorphine is indicated for long-lasting painful conditions requiring an opioid analgesic, e.g. cancer pain. It has also
been recommended for premedication, postoperative pain, in myocardial infarction and in the treatment of morphine
dependence.
Buprenorphine is not suitable for use during labour, because if respiratory depression occurs in the neonate, it cannot be
effectively reversed by naloxone.

Biochemistry
160. Conversion of methylmalonic acid into Succinyl CoA requires
a. Vit B12
b. Folic acid
c. Both (a) and( b)
d. None of the above

Ans: a
Ref: Satyanarayan Biochemistry, 3rd edition, Page No: 155
The degradation of odd chain fatty acids, certain amino acids (valine, Isoleucine etc.) and pyrimidines (thymine and uracil)
produce directly or through the mediation of propionyl CoA, an important compound methylmalonyl CoA. This is
converted by the enzyme methylmalonyl CoA mutase to succinyl CoA in the presence of B12n coenzyme, deoxyadenosyl
cobalamin . This reaction involves hydrogen transfer and intramolecular rearrangement. In B12 deficiency, methylmalonyl
CoA accumulates and is excreted in urine as methylmalonic acid.

www.onlinepgnepal.com Page 81
Online PG Nepal QAE Kartik 5

161. Which of the following determines the class and function of an immunoglobulin?
a. Heavy chain
b. Light chain
c. Variable region
d. Constant region

Ans: a
Ref: Satyanarayan Biochemistry, 3rd edition, Page No: 188
Humansns have five classes of immunoglobulins
immunoglobulins-namely IgG, IgA, IgM, IgD and IgE-containing
containing the heavy chains ϒ, α, μ, δ and
ε, respectively. The type of heavy chain ultimately determines the class and the function of a given lg. Two types of light
chains-namely kappa (κ) and lambda (λ)-areare found in immunoglobulins. They differ in their structure in CL regions. An
immunoglobulin (of any class) contains two K or two λ, light chains and never a mixture. The occurrence of κ chains is
more common in human immunoglobulins ns than λ chains.

162. Which of the following is an erythropoietic porphyria?


a. Acute intermittent porphyria
b. Porphyria cutanea tarda
c. Variegate porphyria
d. Protoporphyria

Ans: d

www.onlinepgnepal.com Page 82
Online PG Nepal QAE Kartik 5

Ref: Satyanarayan Biochemistry, 3rd edition, Page No: 213

163. The least RQ (respiratory


respiratory quotient) is seen in the metabolism of
a. Carbohydrate
b. Protein
c. Fats
d. Alcohol

Ans: d
Ref: Harper Biochemistry 30th Edition, Page No: 148

164. The enzyme glucose phosphatase is present in


a. Muscle
b. Brain
c. Both (a) and (b)
d. None of the above

Ans: d
Ref:
ef: Satyanarayan Biochemistry, 3rd edition, Page No: 266
The fate of glucose 6-phosphate
phosphate depends on the tissue. The liver, kidney and intestine contain the enzyme glucose 6- 6
phosphatase that cleaves glucose 6-phosphate
phosphate to glucose. This enzyme is absent in muscle and brain, hence free glucose
cannot be produced from glucose 6-phosphate
phosphate in these tissues. Therefore, liver is the major glycogen storage organ to
provide glucose into the circulation to be utilized by various tissues.

www.onlinepgnepal.com Page 83
Online PG Nepal QAE Kartik 5

165. Essential fructosuria is due to


a. Deficiency of Fructokinase
b. Deficiency of Aldolase B
c. High consumption of fructose
d. Any of the above

Ans: a
Ref: Satyanarayan Biochemistry, 3rd edition, Page No: 280
Essential fructosuria:
Due to the deficiency of the enzyme hepatic fructokinase, fructose is not converted to fructose 1 -phosphate. This is an
asymptomatic condition with excretion of fructose in urine. Treatment involves the restriction of dietary fructose.
Hereditary fructose intolerance:
This is due to the absence of the enzyme aldolase
ldolase B. Hereditary fructose intolerance causes intracellular accumulation of
fructose .l -phosphate,
phosphate, severe hypoglycemia, vomiting, hepatic failure and jaundice. Fructose 1-phosphate
1 allosterically
inhibits liver phosphorylase and blocks glycogenolysis leading to hypoglycemia. Early detection and intake of diet free
from fructose and sucrose, are advised to overcome fructose intolerance.

166. Overflow proteinuria is seen in


a. Multiple myeloma
b. Nephrotic syndrome
c. Fanconi syndrome
d. Nephrotoxicity due to drugs

Ans: a
Ref: Harper Biochemistry 30th Edition, Page No: 597

167. Which of the following enzyme is involved in the reversible reaction of gluconeogenesis?
a. Hexokinase
b. Phosphofructokinase
c. Pyruvate kinase

www.onlinepgnepal.com Page 84
Online PG Nepal QAE Kartik 5

d. Phosphoglycerate kinase

Ans: d
Ref: Satyanarayan Biochemistry, 3rd edition, Page No: 259
Gluconeogenesis closely resembles the reversed pathway of glycolysis, although it is not the complete reversal of
glycolysis. Essentially, 3 (out of 10) reactions of glycolysis are irreversible. The seven reactions are common for both
glycolysis and gluconeogenesis. The three irreversible steps of glycolysis are catalyzed by the enzymes, namely hexokinase,
phosphofructokinase and pyruvate kinase. These three stages-bypassed by alternate enzymes specific to gluconeogenesis.

Microbiology
168. What is not used in Gram’s stain?
a. Crystal violet
b. Gram’s iodine
c. 30% acetone-alcohol
d. 3% acid-alcohol

Ans: d
Ref: Jawetz’s Medical Microbiology; Lange; 27th Edition, Page No: 743
Gram stain
 Fix smear using heat or methanol.
 Cover with crystal violet stain (10–30 seconds).
 Rinse with water. Do not blot.
 Counterstain with Gram’s iodine stain (10–30 seconds).
 Rinse with water. Do not blot.
 Decolorize with gentle agitation in 30% acetone-alcohol (10–30 seconds, until stain no longer flows off slide).
 Rinse with water. Do not blot.
 Cover with safranin stain (10–30 seconds).
 Rinse with water and air or blot dry.

Ziehl-Neelsen acid-fast stain


 Fix smear using heat.
 Cover with carbolfuchsin, stain gently for 5 minutes over direct flame (or for 20 minutes over a water bath). Do
not permit slides to boil or dry out.
 Rinse with deionized water.
 Decolorize in 3.0% acid-alcohol(95% ethanol and 3.0% hydrochloric acid) until only a faint pink color remains.
 Rinse with water.
 Counterstain for 1 minute with Löffler’s methylene blue stain.
 Rinse with deionized water and let dry.

169. Buruli ulcer is caused by:


a. M. avium
b. M. marinum
c. M. ulcerans
d. M. xenopi

www.onlinepgnepal.com Page 85
Online PG Nepal QAE Kartik 5

Ans: c
Ananthanarayan Microbiology 10th Edition, Page No: 368 - 369
 M. ulcerans (Buruli ulcer): This was originally isolated from human skin lesions in Australia (1948) but have
subsequently been recovered from similar lesions from Uganda (Buruli ulcer), Congo, Nigeria, Mexico,
Malaysia and New Guinea.
 Ulcers are usually seen on the legs or arms and are believed to follow infection through minor injuries. After
an incubation period of a few weeks, indurated nodules appear, which break down, forming indolent ulcers
which slowly extend under the skin.
 Initially, smears from the edge of the ulcer show large clumps of bacilli which are acid fast and alcohol fast.
Later, the immunoreactive phase sets in and the bacilli disappear. The ulcers then heal with disfiguring scars.
 A toxin produced by M. ulcerans causes inflammation and necrosis when injected into the skin of guinea pigs.
This is the only known instance of a toxin produced by any mycobacterium species.
 M. marinum causes a warty skin lesion (swimming pool or fish tank granuloma).

170. Examination of eggs in urine are useful for diagnosis of:


a. Schistosoma hematobium
b. Schistosoma japonicum
c. Schistosoma mansoni
d. None of the above

Ans: a
Ref: Jawetz’s Medical Microbiology; Lange; 27th Edition, Page No: 735 – 736
The Schistosoma worms live in venous system of:
o Schistosoma mansoni: inferior mesenteric veins of large intestine
o Schistosoma japonicum: inferior and superior mesenteric veins of small intestine
o Schistosoma haematobium: veins of urinary bladder
 With S haematobium infections, there is urinary tract involvement: urethral pain, increased urinary frequency,
dysuria, hematuria, and bladder obstruction leading to secondary bacterial infections.
 Diagnosis is by O&P: S mansoni (lateral spine) and S japonicum (barely visible nubby spine) eggs in stool; S
haematobium (terminal spine) eggs in urine.

171. Incorrect about primary immune response is?


a. IgM predominates
b. Antibody affinity is lower
c. Peak response time is 3 – 5 days
d. None of the above

Ans: c
Ref: Ananthanarayan Microbiology, 10th edition, page 148
Differences between primary & secondary immune response:
Features Primary response Secondary response
Lag period after Usually 4 – 10 days Usually 1 – 3 days
immunization
Involving B cell Naïve b cell Memory B cell

www.onlinepgnepal.com Page 86
Online PG Nepal QAE Kartik 5

Peak response time 7 – 10 days 3 – 5 days


Peak antibody response Depends on antigen Usually 100 – 1000 times
more than primary
response
Isotype generated In early response IgM IgG predominates
predominates
Antigens Both thymus dependent & Only thymus dependent
independent
Antibody affinity Lower Higher
Immunization needed Comparatively higher dose Adjuvants not required
of antigens, favorably with
adjuvants (protein antigens)

172. Natural killer cells and Toll-like receptors (TLRs) are mainly classified as components of:
a. Innate immunity
b. Adaptive immunity
c. Humoral immunity
d. Specific immune response

Ans: a > b
Ref: Jawetz’s Medical Microbiology; Lange; 26th Edition, Page No: 123-124
Innate immunity is an immediate response to the pathogen that does not confer long-lasting protective immunity. It is a
nonspecific defense system and includes barriers to infectious agents, such as the skin (epithelium) and mucous
membranes.
It also includes many immune components important in the adaptive immune response, including phagocytic cells, natural
killer (NK) cells, Toll-like receptors (TLRs), cytokines, and complement.

173. The process by which a bacteria stick to host cells is:


a. Adherence
b. Infection
c. Conjugation
d. Pathogenicity

Ans: a
Ref: Jawetz’s Medical Microbiology; Lange; 26th Edition, Page No: 149
 Adherence (adhesion, attachment): The process by which bacteria stick to the surfaces of host cells. After
bacteria have entered the body, adherence is a major initial step in the infection process. The terms adherence,
adhesion, and attachment are often used interchangeably.
 Infection: Multiplication of an infectious agent within the body. Multiplication of the bacteria that are part of the
normal fl ora of the gastrointestinal tract, skin, and so on is generally not considered an infection; on the other
hand, multiplication of pathogenic bacteria (eg, Salmonella species)—even if the person is asymptomatic—is
deemed an infection.
 Pathogenicity: The ability of an infectious agent to cause disease.

174. All of the following are true about Chlamydia except:

www.onlinepgnepal.com Page 87
Online PG Nepal QAE Kartik 5

a. Gram positive
b. Reticulate body is non-infectious
c. Causative organism of psittacosis
d. Elementary body is metabolically inactive

Ans: a
Ref: Surinder Kumar Microbiology, 1st Edition, Page No: 491 491-493
• Chlamydiaceae is a family of obligate intracellular bacterial parasites, small, non motile and gram-negative with a
non-motile
tropism
pism for columnar epithelial cells lining the mucous membranes. They are widely distributed in nature and are
responsible for a variety of ocular, genitourinary and respiratory diseases in man. There is some evidence that
they may be involved in atherosclerosis rosis and, possibly, other chronic diseases.
• They cause psittacosis, lymphogranuloma venarum (LGV) and trachoma in man and diverse diseases in birds and
mammals. Based on the human diseases they were then known to cause, they were called psittacosis
lymphogranuloma-trachoma
trachoma (PLT) viruses, or non committally as ‘PLT agents’ or TRIC (trachoma-inclusion
(trachoma
conjunctivitis) organisms. In recognition of the pioneering studies of Sir Samuel Bedson on psittacosis, the name
Bedsonia was proposed for this group. However However,, the official term for this group now is Chlamydia. The name
Chlamydia is derived from the characteristic appearance of inclusion body by these agents which enclose the
nuclei of infected cells as a cloak or mantle (chlamys, meaning mantle).
Morphology
• Chlamydiae are small, non-motilemotile bacteria. Although they stain poorly with gram’s stain they have the typical LPS
of Gram-negative
negative bacteria. There are two morphologically distinct forms of chlamydiae: elementary body (EB) and
reticulate body (RB).
Elementary body (EB)
• It is a spherical particle, 200-300300 nm in diameter, with a rigid trilaminar cell wall similar to the cell walls of gram-
gram
negative bacteria, and an electron dense nucleoid. The elementary body is the extracellular, infective form,
responsible for attaching to the target host cell and promoting its entry.
Reticulate Body (RB)
• The reticulate body is the intracellular growing and replicative form, 500 500-1000
1000 nm in size (larger than the EB) and
is osmotically fragile. Its cell wall is fragile and pliab
pliable,
le, leading to pleomorphism. They are metabolically active, so
their cytoplasm is rich in ribosomes, which are required for protein synthesis.

175. All of the following statements are true about Bacteriophage except:
a. Phages exhibit two different types of life cycle ,lytic cycle and lysogenic cycle
b. It helps in transduction of bacteria
c. It imparts toxigenicity to bacteria
d. It is bacteria that infects virus

www.onlinepgnepal.com Page 88
Online PG Nepal QAE Kartik 5

Ans: d
Ref: Surinder Kumar Microbiology, 1st Edition, Page No: 529-532
Viruses that infect bacteria are called bacteriophages, or simply phages.
ROLE OF BACTERIOPHAGES
 They play an important role in the transmission of genetic information from one bacterium to another by the
process of transduction.
 They also play a role in the evolution of bacterial types and in the transmission of some virulence characters.
 Phages may indeed be effective in treating bacterial infections, including those caused by antibiotic resistant
bacteria.
 Phages have been used as cloning vectors in genetic manipulations.
 They may have a role in the control of bacterial populations in natural waters.
LIFE CYCLE
 Phages exhibit two different types of life cycle. In the virulent or lytic cycle, intracellular multiplication of the
phage culminates in the lysis of the host bacterium and the release of progeny virions. In the temperate or
lysogenic cycle the phage DNA becomes integrated with the bacterial genome, replicating synchronously with it,
causing no harm to the host cell.
Temperate Phage
 Transduction: Bacteriophages may act as carriers of genes from one bacterium to another. This is known as
transduction. Two types of transduction are recognized: generalized transduction, in which any portion of the
donor DNA can be transferred, and specialized transduction, in which only a specific set of genes can be carried to
a recipient cell.
 Toxin Production: Toxin production in Corynebacterium diphtheriae and Clostridium botulinum types C and D are
determined by genes carried in prophage DNA.
 Antigenic Property: A wide variety of temperate phages of Salmonella can modify the antigenic properties of
somatic O antigen. Such acquisition of new properties by bacterial cells following phage infection is called “phage
conversion”.
 Cloning Vector: Bacteriophages have been used as cloning vectors in genetic manipulations

Community Medicine
176. What is the father of public health?
a. Cholera
b. Polio
c. Small pox
d. Typhoid

Ans: a
Ref: Park's Textbook of Preventive and Social Medicine; 23rd Edition; Page No: 5
 Cholera which is often called the "father of public health" appeared time and again in the western world during
the 19th century.
 An English epidemiologist, John Snow, studied the epidemiology of cholera in London from 1848 to 1854 and
established the role of polluted drinking water in the spread of cholera.

177. What type of disinfection is proper disposal of faeces?

www.onlinepgnepal.com Page 89
Online PG Nepal QAE Kartik 5

a. Concurrent
b. Precurrent
c. Preconcurrent
d. Terminal

Ans: a
Ref: Park's Textbook of Preventive and Social Medicine; 23rd Edition; Page No: 127
Types of disinfection are:
 Concurrent disinfection: It is the application of disinfective measures as soon as possible after the discharge of
infectious material from the body of an infected person, or after the soiling of articles with such infectious
discharges. In other words, the disease agent is destroyed as soon as it is released from the body, and in this way
further spread of the agent is stopped. Concurrent disinfection consists of usually disinfection of urine, faeces,
vomit, contaminated linen, clothes, hands, dressings, aprons, gloves, etc throughout the course of an illness.
 Terminal disinfection: It is the application of disinfective measures after the patient has been removed by death or
to a hospital or has ceased to be a source of infection or after other hospital isolation practices have been
discontinued. Terminal disinfection is now scarcely practised; terminal cleaning is considered adequate, along with
airing and sunning of rooms, furniture and bedding.
 Precurrent (prophylactic) disinfection: Disinfection of water by chlorine, pasteurization of milk and handwashing
may be cited as examples of precurrent disinfection.

178. Which mode of transmission of leptospira is rare?


a. Contact of the broken skin with soil, water or vegetation contaminated by urine of infected animals
b. Direct contact with urine or tissue of infected animals through intact mucus membrane
c. Direct man to man infection
d. Droplet infection when milking infected cows or goats

Ans: c
Ref: Park's Textbook of Preventive and Social Medicine; 23rd Edition; Page No: 292
Mode of transmission of leptospira:
 Direct contact: Leptospira can enter the body through skin abrasions or through intact mucous membrane by
direct contact with urine or tissue of infected animal.
 Indirect contact: Through the contact of the broken skin with soil, water or vegetation contaminated by urine of
infected animals or through ingestion of food or water contaminated with leptospirae.
 Droplet infection: Infection may also occur through inhalation as when milking infected cows or goats by breathing
air polluted with droplets of urine.
 Direct man to man infection is rare.

179. What is the incubation period of chickenpox?


a. 2 to 5 days
b. 8 to 12 days
c. 14 to 16 days
d. 15 to 20 days

Ans: c
Ref: Park's Textbook of Preventive and Social Medicine; 23rd Edition; Page No: 144

www.onlinepgnepal.com Page 90
Online PG Nepal QAE Kartik 5

Incubation period of chickenpox is usually 14 to 16 days, although extremes as wide as 10 to 21 days have been reported.

180. Transmission of filariasis is an example of:


a. Propagative transmission
b. Cyclical transmission
c. Cyclo-developmental transmission
d. Cyclo-propagative transmission

Ans: c
Ref: Park's Textbook of Preventive and Social Medicine; 23rd Edition; Page No: 98
Life cycle of microfilaria in mosquito is cyclo developmental whereas the of malarial parasite is cyclo propagative while for
plague & yellow fever it is propagative.
Biological transmission:
The infectious agent undergoing replication or development or both in vector and requires an incubation period before
vector can transmit.
Biological transmission is of three types:
 Propagative: The agent merely multiplies in vector, but no change in form, e.g., plague bacilli in rat fleas.
 Cyclo-propagative: The agent changes in form and number, e.g., malaria parasites in mosquito.
 Cyclo-developmental: The disease agent undergoes only development but no multiplication, e.g., microfilaria in
mosquito.

181. Screening test has the following features except:


a. Done on apparently healthy individuals
b. It is less accurate
c. Test results are arbitrary and final
d. It can be used as a basis for treatment

Ans: d
Ref: Park's Textbook of Preventive and Social Medicine; 23rd Edition; Page No: 135
Principles of Screening (WHO): Suitability of a Disease for Screening (Criteria)
 The disease should be an important health problem
 There should be an effective treatment available for the disease
 Facilities for diagnosis and treatment should be available
 There should be a latent or early asymptomatic stage of the disease
 There should be a test or examination for the diagnosis of disease
 The test should be acceptable to the population
 The natural history of the disease should be adequately understood
 There should be an agreed policy on who to treat
 The total cost of finding a case should be economically balanced in relation to medical expenditure as a whole
 Case-finding should be a continuous process, not just a 'once and for all' project

www.onlinepgnepal.com Page 91
Online PG Nepal QAE Kartik 5

182. What is number of live births per 1000 women in the reproductive age-group (15-44 or 49 years) in a given year
also known as?
a. Age specific fertility rate
b. General fertility rate
c. General marital fertility rate
d. Gross reproduction rate

Ans: b
Ref: Park's Textbook of Preventive and Social Medicine; 23rd Edition; Page No: 488 – 489
 Birth rate is the simplest indicator of fertility and is defined as "the number of live births per 1000 estimated mid-
year population, in a given year".
 General fertility rate is the "number of live births per 1000 women in the reproductive age-group (15-44 or 49
years) in a given year".
 General marital fertility rate is the "number of live births per 1000 married women in the reproductive age group
(15-44 or 49) in a given year".
 Age-specific fertility rate, defined as the "number of live births in a year to 1000 women in any specified age-
group".
 Age-specific marital fertility rate is the number of live births in a year to 1000 married women in any specified age
group.
 Total fertility rate represents the average number of children a woman would have if she were to pass through her
reproductive years bearing children at the same rates as the women now in each age group. It is computed by
summing the age-specific fertility rates for all ages; if 5-year age groups are used, the sum of the rates is multiplied
by 5.
 Total marital fertility rate is the average number of children that would be born to a married woman if she
experiences the current fertility pattern throughout her reproductive span.

www.onlinepgnepal.com Page 92
Online PG Nepal QAE Kartik 5

 Gross reproduction rate is the average number of girls that would be born to a woman if she experiences the
current fertility pattern throughout her reproductive span (15(15-44 or 49
9 years), assuming no mortality.
 Net Reproduction Rate (NRR) is defined as the number of daughters a newborn girl will bear during her lifetime
assuming fixed age-specific
specific fertility and mortality rates.

183. What does “d/(c + d) x 100” represent?


a. Predictive value of a negative test
b. Predictive value of a positive test
c. Percentage of false-negatives
d. Percentage of false-positive

Ans: a
Ref: Park's Textbook of Preventive and Social Medicine; 23rd Edition; Page No: 139

 The letter "a" above denotes those indiv


individuals
iduals found positive on the test who have the condition or disorder being
studied (i.e., true-positives).
 The group labelled "b" includes those who have a positive test result but who do not have the disease {i.e., false-
false
positives).
 Group "c" includes those
hose with negative test results but who have the disease (i.e., false
false-negatives).
negatives).
 Finally, those with negative results who do not have the disease are included in group "d" (i.e., true-negatives).
true

Evaluation of a screening test: The following measures aare used to evaluate a screening test:
 Sensitivity a/ (a + c) x 100
 Specificity = d/(b + d) x 100
 Predictive value of a positive test = a/(a + b) x 100
 Predictive value of a negative test = d/(c + d) x 100
 Percentage of false-negatives
negatives = c/(a + c) x 10
100
 Percentage of false-positive
positive = b/(b + d) x 100

184. What is incorrect about precision of a test?


a. Precision is the consistency and reproducibility of a test.
b. Random error decreases precision in a test.
c. Increased precision is associated with an increased standard deviation.
d. Increased precision increases statistical power of a test.

Ans: c
Ref: USMLE Step 1 First Aid 2020
 Precision (reliability) is the consistency and reproducibility of a test. It is the absence of random variation in a test.
Random errorr decreases precision in a test; whilst increased precision is associated with a decreased standard
deviation and an increased statistical power of a test.
 Accuracy (validity) is the trueness of test measurements. It is the absence of systemic error or bias bia in a test.
Systemic error decreased the accuracy in a test.

www.onlinepgnepal.com Page 93
Online PG Nepal QAE Kartik 5

185. What is an estimate of how much variability exists in a (theoretical) set of sample means around the true
population mean known as?
a. Mean deviation
b. Median deviation
c. Standard deviation
d. Standard error

Ans: d
Ref: USMLE Step 1 First Aid 2020
 Standard deviation = how much variability exists in a set of values, around the mean of these values.
 Standard error = an estimate of how much variability exists in a (theoretical) set of sample means around the true
population mean.

Forensic Medicine
186. Feature of human hair are all EXCEPT:
a. Cuticular scales are short and flattened
b. Medulla is continuous and wide
c. Cortex is thick and medulla is one third of shaft diameter
d. Pigment in medulla is light and
nd broken

Ans: b
Ref: Gautam Biswas Forensic Medicine, 3rd Edition, Page No: 90

187. Barbiturates excretion in urine may be increased by:


a. Diuretics
b. Alkalinization
c. Acidification
d. Dialysis

Ans: b
Barbiturates & salicylates excretion ↑ by alkalinizanon of urine.

www.onlinepgnepal.com Page 94
Online PG Nepal QAE Kartik 5

188. Medical treatment related clauses are mentioned in which chapter of Civil and Criminal Code Act 2074?
a. Chapter 11
b. Chapter 13
c. Chapter 17
d. Chapter 19

Ans: d
Civil and Criminal Code Act 2074 came into effect from Bhadra 1, 2075. Wednesday. Parliament brought the new code in
order to update the criminal law which was earlier based on the amendment of Civil Code-2021. Legislature-Parliament
meeting has passed the long-awaited Criminal Code 2074 on Asoj 30, 2074.
Various Sections:
 5: Social welfare, Health, Security
 13: Abortion related crime
 14: Injury related crime
 18: Rape related crime
 19: Medical treatment related

189. Medical etiquette is related to:


a. Legal responsibilities of doctors
b. Rules to be followed by doctors
c. Courtesy observed between doctors
d. Moral principles followed by doctors

Ans: c
Ref: Gautam Biswas Forensic Medicine, 3rdEdition, Page No: 3
Forensic medicine* (Legal medicine or State medicine): It is the application of principle and knowledge of medical
sciences to legal purposes and legal proceedings so as to aid in the administration of justice.
Medical jurisprudence (Latin juris: law, prudentia: knowledge or skill): It is the application of knowledge of law in relation
to practice of medicine. It includes:
 Doctor-patient relationship
 Doctor-doctor relationship
 Doctor-State relationship.

Medical etiquette: These are the conventional laws and customs of courtesy which are followed between members of
same profession.
A doctor should behave with his colleagues, as he would like to have them behave with him, e.g. he should not charge
another doctor or members of his family for professional service.
Medical ethics: It is concerned with moral principles for the members of the medical profession in their dealings with each
other, their patients and the State. It is a self-imposed code of conduct assumed voluntarily by medical professionals.

190. Antemortem bruise is differentiated from postmortem bruise by:


a. Well-defined margin
b. Capillary rupture with extravasation of blood
c. Yellow color

www.onlinepgnepal.com Page 95
Online PG Nepal QAE Kartik 5

d. Gaping

Ans: b
Ref: Gautam Biswas Forensic Medicine, 3rd Edition, Page No: 194
Difference between Antemortem and postmortem bruise
Feature Antemortem bruise Postmortem bruise
Swelling Present Absent
Damage to epithelium Present Absent
Extravasation of blood Present Absent
Coagulation Present Absent
Infiltration of the tissues Present Absent
with blood
Color changes Seen Uniform color
Margins Merge with surrounding area Sharply demarcated
Appearance More marked in victims who survive Less marked
for sometime

Mandatory CPD Topics


191. In disaster impact and response, the greatest need for emergency care occurs in:
a. First few hours
b. First few days
c. First few months
d. First few years

Ans: a
 Medical treatment for large number of casualties is likely to be needed only after certain types of disaster.
 Most injuries are sustained during the impact, and thus, the greatest need for emergency care occurs in the first
few hours.
 The management of mass casualties can be further divided into search and rescue, first aid, triage and stabilization
of victims, hospital treatment and redistribution of patients to other hospitals if necessary.

192. You suspected the symptoms of COVID 19 in a patient with recent travel history to COVID 19 endemic area. You
want to admit the patient in isolation ward and investigate further. But patient wants to go away. What will you do?
a. Let the patient go
b. Counsel the patients about the health hazard to the patient and community
c. Call police officers and get admitted anyway
d. Ask the patient to stay in follow up

Ans: b
Counsel the patient, give details about the condition and make him realize the significance of getting isolated and
investigated.

193. Which of the following is effective communication skill in patient care?


a. Use silence effectively, allowing patient enough time to express thoughts or feelings

www.onlinepgnepal.com Page 96
Online PG Nepal QAE Kartik 5

b. Actively encourage patient through use of supportive words or comments


c. Use open, exploratory questions – inviting patient to become actively involved
d. All of the above

Ans: d
Communication is the major part of medicine
 Good communication skills are integral to medical and other healthcare practice.
 Communication is important not only to professional- patient interaction but also within the healthcare team.
 The benefits of effective communication include good working relationships and increased patient satisfaction.
 Effective communication may increase patient understanding of treatment, improve compliance and, in some
cases, lead to improved health
Communication skills some techniques
 Developed fluent dialogue with patient
 Used silence effectively, allowing patient enough time to express thoughts or feelings
 Actively encouraged patient through use of supportive words or comments
 Dialogue with patient was enhanced by effective use of non-verbal behaviour
 Used open, exploratory questions – inviting patient to become actively involved
 Adjusted language as appropriate, to suit particular needs of the situation

194. Which of the following is not AHA guideline for patients with suspected or confirmed COVID-19?
a. Aerosol-generating procedures such as CPR and endotracheal intubation expose providers to a greater risk of
disease transmission and should be performed in airborne infection isolation rooms (AIIRs); personnel should use
respiratory protection.
b. Patients with known or suspected COVID-19 should be cared for in a single-person room with the door closed and
AIIRs should be reserved for patients undergoing aerosol-generating procedures.
c. N95 respirators or respirators that offer a higher level of protection should be used instead of a face mask during
aerosol-generating procedures.
d. Use high-flow nasal oxygenation and mask CPAP or bilevel CPAP due to lesser risk of aerosol generation.

Ans: d
The AHA advises standard and transmission-based precautions be used when caring for patients with suspected or
confirmed COVID-19. Among the specific guidance included in the three-page document:
 Aerosol-generating procedures such as CPR and endotracheal intubation expose providers to a greater risk of
disease transmission and should be performed in airborne infection isolation rooms (AIIRs); personnel should use
respiratory protection.
 Only providers essential for patient care and procedural support should be present during the procedure and the
room should be cleaned and disinfected following the procedure.
 Patients with known or suspected COVID-19 should be cared for in a single-person room with the door closed and
AIIRs should be reserved for patients undergoing aerosol-generating procedures.
 N95 respirators or respirators that offer a higher level of protection should be used instead of a face mask during
aerosol-generating procedures.
 Providers should put on a respirator or facemask (if a respirator is not available) before entering a patient's room
or care area. Facilities should return to use of respirators for patients with known or suspected COVID-19 once the
supply chain is restored.
 Wear eye protection, gloves, and gowns.

www.onlinepgnepal.com Page 97
Online PG Nepal QAE Kartik 5

 When gowns are in short supply, they should be prioritized for aerosol-generating procedures, care activities
where splashes and sprays are anticipated, and high-contact patient care activities that provide opportunities for
transfer of pathogens to the hands and clothing of providers.
 If intubation is needed, consider using rapid sequence intubation with appropriate personal protective equipment
(PPE).
 If possible, avoid procedures which generate aerosols, such as bag-valve mask, nebulizers and non-invasive
positive pressure ventilation.
Consider proceeding directly to endotracheal intubation in patients with acute respiratory failure. Avoid the use of high-
flow nasal oxygenation and mask CPAP or bilevel CPAP due to greater risk of aerosol generation.

195. In adults, while doing CPR, chest should be compressed:


a. Upto 2 inches
b. At least 2 inches
c. Maximum 2 inches
d. Average 2 inches

Ans: b
Chest compression
 The provider should do the following:
 Place the heel of one hand on the patient’s sternum and the other hand on top of the first, fingers interlaced
 Extend the elbows and the provider leans directly over the patient (see the image below)
 Press down, compressing the chest at least 2 in
 Release the chest and allow it to recoil completely
 The compression depth for adults should be at least 2 inches (instead of up to 2 inches, as in the past)
 The compression rate should be at least 100/min
 The key phrase for chest compression is, “Push hard and fast”
 Untrained bystanders should perform chest compression–only CPR (COCPR)
 After 30 compressions, 2 breaths are given; however, an intubated patient should receive continuous
compressions while ventilations are given 8-10 times per minute
 This entire process is repeated until a pulse returns or the patient is transferred to definitive care
 To prevent provider fatigue or injury, new providers should intervene every 2-3 minutes (ie, providers should swap
out, giving the chest compressor a rest while another rescuer continues CPR

196. Commonly used drug for surgical prophylaxis is:


a. Penicillin, vancomycin, and metronidazole
b. Penicillin, Aztreonam, and gentamicin
c. Cefazolin, aztreonam, and metronidazole
d. Cefazolin, vancomycin, and gentamicin

Ans: d
The three antibiotics used in adult surgical prophylaxis where weight-based dosing is recommended are cefazolin,
vancomycin, and gentamicin. For patients receiving cefazolin, 2 g is the current recommended dose except for patients
weighing greater than or equal to 120 kg, who should receive 3 g.

197. Which of the following is definition of “Contact” for COVID-19?

www.onlinepgnepal.com Page 98
Online PG Nepal QAE Kartik 5

a. Face-to-face contact with a probable or confirmed case within 1 meter and for more than 15 minutes
b. Direct physical contact with a probable or confirmed case
c. Direct care for a patient with probable or confirmed COVID-19 disease without using proper personal protective
equipment
d. All of the above

Ans: d
Definition of contact in COVID-19:
A contact is a person who experienced any one of the following exposures during the 2 days before and the 14 days after
the onset of symptoms of a probable or confirmed case:
1. Face-to-face contact with a probable or confirmed case within 1 meter and for more than 15 minutes;
OR
2. Direct physical contact with a probable or confirmed case;
OR
Direct care for a patient with probable or confirmed COVID-19 disease without using proper personal protective
equipment;
OR
3. Other situations as indicated by local risk assessments. Note: for confirmed asymptomatic cases, the period of contact is
measured as the 2 days before through the 14 days after the date on which the sample was taken which led to
confirmation.

198. What questions will you ask to screen patents for possible COVID-19?
a. Do you have Cough? Fever? Shortness of breath?
b. Did you recently return from, travel in, or been living in, an affected area in the past 2 weeks?
c. Have you been in close contact in the past 2 weeks with someone returning from an affected area?
d. All of the above

Ans: d
Screening questionnaire for COVID-19:
All individuals presenting to the OPD or ER entrance should be screened with the following questions:
a. Symptoms: Do you have any of the following symptoms?
 Cough? Fever? Shortness of breath? (common)
 Sore throat, headache or body ache? (less common)
b. Travel history or contact with traveler: Have you?
 Recently returned from, travel in, or been living in, an affected area in the past 2 weeks?
 Been in close contact in the past 2 weeks with someone returning from an affected area?
c. Exposures: Did you have any exposures to any of the following in last 2 weeks?
 Close contact with anyone with fever or respiratory illness of unknown cause
 Known or suspected COVID-positive contact

199. Which step is not a part of the five steps in the Adult Chain of Survival?
a. Early CPR
b. Rapid defibrillation
c. Advanced airway placement
d. Integrated post-cardiac arrest care

www.onlinepgnepal.com Page 99
Online PG Nepal QAE Kartik 5

Ans: c
 Early CPR provides vital oxygen to the brain and vital organs increasing the likelihood of recovery.
 Rapid defibrillation increases the chance of effectively restoring a normal heart rhythm.
 Advanced airway placement is not part of the BLS adult chain of survival.
 Post cardiac arrest care reduces the possibility of long-term impairment and increases the chance of a victim
making a full recovery.

200. Damage control resuscitation consists of:


a. Permissive hypotension
b. Minimizing crystalloid-based resuscitation
c. Administration of balanced ratios of blood products
d. All of the above

Ans: d
Ref: Schwartz’s Principles of Surgery, Eleventh Edition, Page No: 104
Damage control resuscitation has three basic components: permissive hypotension, minimizing crystalloid-based
resuscitation, and the administration of balanced ratios of blood products.

NEXT MD/MS BASED ENTRANCE


07:00 PM - 09:00 PM, Kartik 12, 2077 (Wednesday evening): Mandatory CPD Topics Based (100 marks)
11:00 AM - 02:00 PM, Kartik 15, 2077 (Saturday noon): CEE (200 marks) MD/MS Based
07:00 PM - 09:00 PM, Kartik 17, 2077 (Monday evening): Subjectwise [Gynae/Obs, ENT, Ophthalmology] (100 marks)

Visit our Website for Online Test: www.onlinepgnepal.com

BEST WISHES

www.onlinepgnepal.com Page 100

Вам также может понравиться